Sie sind auf Seite 1von 72

JEE (MAIN & ADV.

), MEDICAL + BOARD, NDA, X & IX


Enjoys unparalleled reputation for best results
in terms of percentage selection
www.newtonclasses.net

MATRICES ( )
Only one option is correct.
1. The number of 3 × 3 non-singular matrices, with four entries as 1 and all other entries as 0, is
(a) less than 4 (b) 5 (c) 6 (d) at least 7
2. If A and B are square matrices of size n × n such that A2 − B 2 = ( A − B ) ( A + B ) , then which of the
following will be always true?
(a) A = B (b) AB = BA
(c) either of A or B is zero matrix (d) either of A or B is identity matrix
 x − y − z  0
3. If  − y + z  = 5  , then the values of x, y and z are respectively
 z  3 
(a) 5, 2, 2 (b) 1, –2, 3 (c) 0, –3, 3 (d) 11, 8, 3
(e) 4, 1, 3
4. Let A and B be two symmetric matrices of same order. Then, the matrix AB − BA is
(a) a symmetric matrix (b) a skew-symmetric matrix (c) a null matrix (b) the identity matrix
5. If A is a square matrix, then
(a) A + AT is symmetric (b) AAT is skew-symmetric
(c) AT + A is skew-symmetric (d) AT A is skew-symmetric
 x 1
6. If A =  and A2 is the identity matrix, then x is equal to
1 0

(a) –1 (b) 0 (c) 1 (d) 2


ω 0 
7. If ω is a complex cube root of unity and A =   , then A is
50

 0 ω 
(a) ω 2 A (b) ω A (c) A (d) 0
8. If A and B are square matrices of order 3 × 3, then which of the following is true?
(a) AB = O ⇒ A = O or B = O (b) det ( 2 AB ) = 8det ( A ) det ( B )
(c) A2 − B 2 = ( A + B )( A − B ) (d) det ( A + B ) = det ( A) + det ( B )
9. If X and Y are 2 × 2 matrices such that 2 X + 3Y = O and X + 2Y = I , where O and I denote the
2 × 2 zero matrix and the 2 × 2 identity matrix, then X is equal to
1 0  2 0  −3 0  3 0
(a)  (b)  (c)  (d) 
0 1  0 2  0 − 3 0 3
   

 1 −2   3 6
10. If A =  and f ( t ) = t 2 − 3t + 7, then f ( A ) +   is equal to
4 5   −12 −9 

1 0  0 0 0 1  1 1 
(a)  (b)  (c)  (d) 
0 1  0 0 1 0  0 0
   

1 0 0 
11. The characteristic roots of the matrix  2 3 0 are
 4 5 6 
 

Office.: 606 , 6 Floor, Hariom Tower, Circular Road, Ranchi-1, Ph.: 0651-2562523, 9835508812, 8507613968
th
2 ( ) BY R. K. MALIK’S NEWTON CLASSES
(a) 1, 3, 6 (b) 1, 2, 4 (c) 4, 5, 6 (d) 2, 4, 6
3 3 3
12. If A = 3 3 3 , then A4 is equal to
3 3 3
(a) 27 A (b) 81 A (c) 243 A (d) 729 A
1 2 3   x 
13. If [1 x 1] 0 5 1   1 = 0, then the value of x is
0 3 2   −2 
2 5 4
(a) 0 (b) (c) (d) −
3 4 5
 0 a
4

14.  b 0  = I , then
 
(a) a = 1 = 2b (b) a = b (c) a = b 2 (d) ab = 1
2 −1 4 5 0 3 
15. If 2 A + 3B =  and A + 2 B =   , then B is
3 2 5 1 6 2 

 8 −1 2  8 1 2  8 1 −2  8 1 2
(a)  (b)  (c)  (d) 
 −1 10 − 1  −1 10 − 1  −1 10 − 1  1 10 1 
  

16. If O ( A ) = 2 × 3, O ( B ) = 3 × 2, and O ( C ) = 3 × 3, which one of the following is not defined?


(a) CB + A ' (b) BAC (c) C ( A + B ') ' (d) C ( A + B ')
1
17. If A is square matrix, A ' , is its transpose, then ( A − A ') is
2
(a) a symmetric matrix (b) a skew-symmetric matrix
(c) a unit matrix (d) an elementary matrix
1 2 
18. If f ( x ) = x 2 + 4 x − 5 and A =   , then f ( A) is equal to
 4 − 3
 0 −4  2 1  1 1  8 4 
(a)  (b)  (c)  (d) 
8 8  2 0 1 0  8 0 
   

19. If A and B are square matrices of size n × n such that A2 − B 2 = ( A − B )( A + B ) , then which of the
following will be always true?
(a) AB = BA (b) either of A or B is a zero matrix
(c) either of A or B is an identity matrix (d) A = B
1 2   a 0
20. Let A =  and B =   , a, b ∈ N . Then
3 4  0 b

(a) there exist more than one but finite number of B ' s such that AB = BA
(b) there exists exactly one B such that AB = BA
(c) there exists infinitely many B ' s such that AB = BA
(d) there cannot exist any B such that AB = BA
1 1
21. If A =  , then A100 is equal to
1 1

(a) 2100 A (b) 299 A (c) 100 A (d) 299 A

Office.: 606 , 6 Floor, Hariom Tower, Circular Road, Ranchi-1, Ph.: 0651-2562523, 9835508812, 8507613968
th
( ) BY R. K. MALIK’S NEWTON CLASSES 3
 1 − 3
22. If A =  and A2 − 4 A − 10 I = A, then k is equal to
2 k 

(a) 0 (b) –4 (c) 4 and not 1 (d) 1 or 4


23. If a square matrix A is such that AAT = I = AT A, then A is equal to
(a) 0 (b) ± 1 (c) ± 2 (d) None of these
 cos α sin α 
24. If A (α ) =  , then the matrix A2 (α ) is
 − sin α cos α 

(a) A ( 2α ) (b) A (α ) (c) A ( 3α ) (d) A ( 4α )


25. Let A, B and C be n × n matrices. Which one of the following is a correct statements?
(a) If AB = AC , then B = C (b) If A3 + 2 A2 + 3 A + 5 I = O; then A is invertible
(c) If A2 = O, then A = 0 (d) None of the above
 1 1 
 2 2 
26. The matrix A =   is
 1 1 
− 2 − 2 
 
(a) unitary (b) orthogonal (c) nilpotent (d) involutory
3   2
27. If U = [ 2 − 3 4] , X = [ 0 2 3] , V = 2 and Y =  2  , then UV + XY
 
1   4 
   

(a) [ 20] (b) 20 (c) [ −20] (d) –20


28. If A is a matrix such that A2 = A + I , where I is the unit matrix, then A5 is equal to
(a) 5A + I (b) 5 A + 2 I (c) 5 A + 3I (d) 5 A + 4 I
29. If m [ −3 4] + n [ 4 − 3] = [10 − 11] , then 3m + 7 n is equal to
(a) 3 (b) 5 (c) 10 (d) 1
 1 2 3   −1 −2 
 −4 −5 − 6
30. If P =  2 3 4   −2 0  , then P22 is equal to
0 0 1 
 3 4 6   0 −4 
(a) 40 (b) –40 (c) –20 (d) 20
a g x 
31. If A = [ x y z ] , B =  h f  and C =  y  . Then, ABC = O, if
h
b 
 g f c   z 

(a)  ax 2 + by 2 + cz 2 + 2 gxy + 2 fyz + 2czx  = O (b)  ax 2 + cy 2 + bz 2 + xy + yz + zx  = O

(c)  ax 2 + by 2 + cz 2 + 2hxy + 2by + 2cz  = O (d)  ax 2 + by 2 + cz 2 + 2 gzx + 2hxy + 2 fyz  = O

 6 8 5
32. If A =  4 2 3 is the sum of a symmetric matrix B and skew-symmetric matrix C , then B is
9 7 1

6 6 7   0 2 −2  6 6 7  0 6 −2 
(a)  6 2 5  (b)  −2 5 −2 (c)  −6 2 −5 (d)  2 0 −2 
7 5 1   2 2 0   −7 5 1   −2 −2 0 

Office.: 606 , 6 Floor, Hariom Tower, Circular Road, Ranchi-1, Ph.: 0651-2562523, 9835508812, 8507613968
th
4 ( ) BY R. K. MALIK’S NEWTON CLASSES
α 0  1 0 
33. If A =  and B =  , then value of α for which A2 = B is
 1 1 5 1 
 

(a) 1 (b) –1 (c) 4 (d) No real values


0 1 
34. If A =  , I is the unit matrix of order 2 and a, b are arbitrary constant, then ( aI + bA ) is equal
2

0 0 

to
(a) a 2 I − abA (b) a 2 I + 2abA (c) a 2 I + b 2 A (d) None of the above
 −1 2 5 
35. If the rank of the matrix  2 −4 a − 4  is 1, then the value of a is

 1 −2 a + 1 
(a) –1 (b) 2 (c) –6 (d) 4
 −3   4  10 
36. If x   + y   =   , then
 4 3   −5
(a) x = −2, y = 1 (b) x = −9, y = 10 (c) x = 22, y = 1 (d) x = 2, y = −1
4 1 0 0
37. The rank of the matrix  3 0 1 0 is
 6 0 2 0
(a) 4 (b) 3 (c) 2 (d) 1
2 4 5 
38. If A =  4
 8 10  , the rank of A is equal to
 −6 −12 −15
(a) 0 (b) 1 (c) 2 (d) 3
 1 −1 1 
39. The ranks of  1 1 −1 is
 −1 1 1 
(a) 0 (b) 1 (c) 2 (d) 3
 −1 2 5 
40. The rank of the matrix  2 −4 a − 4  is

 1 −2 a + 1 
(a) 1 if a = 6 (b) 2 if a = 1 (c) 3 if a = 2 (d) 4 if a = −6
 x1 y1 1
41. If the points ( x1 , y1 ) , ( x2 , y2 ) and ( x3 , y3 ) are collinear, then the rank of the matrix  x2 y2 1 will
 x3 y3 1
always be less than
(a) 2 (b) 3 (c) 1 (d) None of these
1 0 1
42. If A =  0 1 1  , then A is :
 1 0 0 
(a) symmetric (b) skew-symmetric (c) non-singular (d) singular

Office.: 606 , 6 Floor, Hariom Tower, Circular Road, Ranchi-1, Ph.: 0651-2562523, 9835508812, 8507613968
th
( ) BY R. K. MALIK’S NEWTON CLASSES 5

 4 6 −1   2 4 3
43. A =  3 0 2  , B =  0 1  , C =  1  , then the expression which is not defined is :
   
 1 −2 5   −1 2   2 
(a) A2 + 2 B − 2 A (b) CC ′ (c) B′C (d) AB
1/ 3 2  3 6 
44. If A =  , B=  and AB = I , then x =
 0 2x − 3   0 −1 

(a) −1 (b) 1 (c) 0 (d) 2


45. If AB = C , then matrices A, B, C are :
(a) A2×3 , B3×2 , C2×3 (b) A3×2 , B2×3 , C3×2 (c) A3×3 , B2×3 , C3×3 (d) A3×2 , B2×3 , C3×3
1 
46. If A =   , then for what value of λ , A = O :
2
λ
 − 1 − λ 
(a) 0 (b) ±1 (c) −1 (d) 1
 0 1 −2 
47. If the matrix  −1 0 3  is singular, then λ =
 λ −3 0 
(a) −2 (b) −1 (c) 1 (d) 2
48. If A is a square matrix of order n and A = kB , where k is a scalar, then A =
(a) B (b) k B (c) k n B (d) n B
 3 −4 
49. If X =  , then the value of X n is :
 1 −1 

 3n −4n   2+n 5−n   3n ( −4 ) 


(a)  (b)  (c)  (d) none of these
n

 1n ( −1) 
 
 n −n   n − n  n

50. Matrix theory was introduced by :


(a) Newton (b) Cayley-Hamilton (c) Cauchy (d) Euclid
51. Which one of the following is not true :
(a) Matrix addition is commutative (b) Matrix addition is associative
(c) Matrix multiplication is commutative (d) Matrix multiplication is associative
3 2
52. If U = [ 2 − 3 4 ] , X = [ 0 2 3 ] , V =  2  and Y =  2  , then UV + XY =
 
 1   4 
(a) 20 (b) [ −20] (c) −20 (d) [ 20]
53. Choose the correct answer :
(a) every identity matrix is a scalar matrix (b) every scalar matrix is an identity matrix
(c) every diagonal matrix is an identity matrix
(d) a square matrix whose each element is 1 is an identity matrix
 7 1 
2 34 17 
54. If A =  and A−1 =   , then the value of x is :
 −  
3 7   −3 2 
x

 34 17 
(a) 2 (b) 3 (c) –4 (d) 4

Office.: 606 , 6 Floor, Hariom Tower, Circular Road, Ranchi-1, Ph.: 0651-2562523, 9835508812, 8507613968
th
6 ( ) BY R. K. MALIK’S NEWTON CLASSES
55. If I is a unit matrix of order 10, then the determinant of I equal to :
(a) 10 (b) 1 (c) 1 /10 (d) 9
α 0  1 0
56. If A =  and B =  , then value of α for which A2 = B , is :
1 1 5 1
 

(a) 1 (b) −1 (c) 4 (d) no real values


1 0 0
57. The inverse of the matrix  0 1 0  is :
 0 0 1 
0 0 1 1 0 0 0 1 0 1 0 0
(a)  0 1 0  (b)  0 1 0  (c)  0 0 1  (d)  0 0 1 
 1 0 0   0 0 1   1 0 0   0 1 0 
3 2
58. If A =   , then A ( adj A) =
1 4
 10 0   0 10   10 1 
(a)  (b)  (c)  (d) none of these
 0 10   10 0   1 10 
  

 4 x+2
59. If A =   is symmetric, then x =
 2x − 3 x +1 
(a) 3 (b) 5 (c) 2 (d) 4
60. If A is a square matrix A + A is symmetric matrix, then A − A =
(a) unit matrix (b) symmetric matrix (c) skew symmetric matrix (d)zero matrix
T T

61. If the multiplicative group of 2 × 2 matrices of the form   , for a ≠ 0 and a ∈ R , then the
a a

2 2
a a
inverse of   is :
2 2
 1/ 8 1 / 8   1/ 4 1/ 4   1/ 2 1/ 2 
(a)  (b)  (c)  (d) does not exist
 1/ 8 1 / 8   1/ 4 1/ 4   1/ 2 1/ 2 
  

 2 −2 −4 
62. If  −1 3 4  is an indempotent matrix, then x is equal to :
 1 −2 x 
 

(a) −5 (b) −1 (c) −3 (d) −4


63. If A is non-singular matrix, then det ( A−1 ) is equal to :
 1  1 1 1
(a) det  2  (b) (c) det   (d)
A  det ( A2 )  A det ( A)
64. If A is a skew-symmetric matrix, then trace of A is :
(a) −5 (b) 0 (c) 24 (d) 9
1 1 i
65. If the matrix A =  , where i = −1 , is unitary, then a is equal to :
2  −i a 

(a) −2 (b) −1 (c) 0 (d) 1


1 2 2 
66. If 3 A =  2 1 −2  and AA′ = I , then x + y is equal to :
 x 2 y 
 

Office.: 606 , 6 Floor, Hariom Tower, Circular Road, Ranchi-1, Ph.: 0651-2562523, 9835508812, 8507613968
th
( ) BY R. K. MALIK’S NEWTON CLASSES 7
(a) −5 (b) −4 (c) −3 (d) −2
67. The sum of two idempotent matrices A and B is idempotent, if AB + BA =
(a) 4 (b) 3 (c) 2 (d) 0
1 2 3
68. The rank of  2 4 6  is equal to :
 3 6 9 
(a) 1 (b) 2 (c) 3 (d) none of these
69. If A + A − I = O , then A is equal to :
2 −1

(a) A − I (b) I − A (c) I + A (d) none of these


70. If A is a square matrix, then adj AT − ( adj A ) is equal to :
T

(a) 2 A (b) 2 A I (c) null matrix (d) unit matrix


71. If A =  aij  is a matrix of rank r , then :

(a) r = min ( m, n ) (b) r > min ( m, n ) (c) r ≤ min ( m, n ) (d) none of these
m× n

72. If A is an orthogonal matrix, then :


(a) A = 0 (b) A = ±1 (c) A = ±2 (d) none of these
73. If A is an orthogonal matrix, then A−1 equals :
(a) A (b) A′ (c) A2 (d) none of these
74. If A is a square matrix of order 2 × 2 such that A = O , then :
2

(a) A =   , where α , β , γ are numbers such that α + βγ = 0


2
α β 
γ −α 

(b) A =   with α = ± β
α β 
β −α 

(c) A =   with α + β = 1
2 2
(d) A =   with α + β = −1
2 2
 α −α   α α
 −β β   −β β 
75. In matrices :
(a) ( A + B ) = A2 + 2 AB + B 2 (b) ( A + B ) = A2 + B 2
2 2

(c) ( A + B ) ≠ A2 + 2 AB + B 2 (d) ( A + B ) = A2 + 2 BA + B 2
2 2

1 0 0 
76. If A =  0 1 0  , then A2 is equal to :
 a b −1 
(a) A (b) − A (c) null matrix (d) I
77. If n order square matrix A is orthogonal, then adj ( adj A ) is :
(a) Always −1 if n is even (b) Always 1 if n is odd
(c) Always 1 (d) Always –1
78. If a, b, c are positive integers and aa′ + bb′ + cc′ = 0 , then the rank of the matrix
 0 − c b − a′ 
0 − a −b′ 
is :
 c
0 − c′ 

 −b a
0 
 
 a ′ b′ c ′

Office.: 606 , 6 Floor, Hariom Tower, Circular Road, Ranchi-1, Ph.: 0651-2562523, 9835508812, 8507613968
th
8 ( ) BY R. K. MALIK’S NEWTON CLASSES
(a) 1 (b) 2 (c) 3 (d) 4
79. If commutativity is not true in any multiplication of 3 × 3 matrices A and B (or their powers), then
the number of distinct terms in the expansion of ( A + B ) must be :
3

(a) 3 (b) 4 (c) 6 (d) 8


80. If A is square matrix of order 3 × 3 such that A = 2 , then −1 −1
( adj A ) is :

(a) 2 (b) 4 (c) 6 (d) 9


a 0
81. Let a and b two real numbers such that a > 1, b > 1 . If A =   , then lim
−1

0 b →∞
( An ) =

(a) Unit matrix (b) Null matrix (c) 2 I (d) none of these
n

82. If A2 = I , then the value of det ( A − I ) is (where A has order 3) : (where A > 0 )
(a) −1 (b) 0 (c) 1 (d) can not say anything
83. If adj B = A, P = Q = 1 , then adj ( Q BP −1 −1
) is :
(a) PQ (b) QAP (c) PAQ (d) PA−1Q
1 2  1+ x 
84. If A =  and f ( x ) =   , then f ( A) is :
2 1  1− x 

1 1 2 2  −1 −1 
(a)  (b)  (c)  (d) none of these
1 1 2 2  −1 −1 
  

1 2
85. Let A =  and  c d  are two matrices such that AB = BA and c ≠ 0 , then value of
a b 
3 4
 B =
 
is :
a−d
3b − c
(a) −2 (b) −1 (c) 0 (d) 1
1 1
86. If both A − I and A + I are orthogonal matrices, then :
2 2
3
(a) A is orthogonal (b) A2 = I
4
(c) A is skew symmetric matrix of even order (d) none of the above
87. If P is orthogonal matrix and Q = PAP′ and X = P′Q 2010 P , then X −1 is (where A is involuntary
matrix) :
(a) A2010 (b) A−2010 (c) A (d) I
 1 
88.  −1  [ 2 1 −1 ] =
 
 2 
 2   2 1 −1 
(a) [ − 1 ] (b)  −1 
  (c)  −2 −1 1  (d) not defined
 −2   4 2 −2 
89. If two matrices A and B are of order p × q and r × s respectively, can be subtracted only, if :
(a) p = q (b) p = q, r = s (c) p = r , q = s (d) none of these
90. If the matrix AB = O , then :
(a) A = O or B = O (b) A = O and B = O
(c) It is not necessary that either A = O or B = O

Office.: 606 , 6 Floor, Hariom Tower, Circular Road, Ranchi-1, Ph.: 0651-2562523, 9835508812, 8507613968
th
( ) BY R. K. MALIK’S NEWTON CLASSES 9
(d) A ≠ O, B ≠ O
91. If d is the determinant of a square matrix A of order n , then the determinant of its adjoint is :
(a) d n (b) d n −1 (c) d n +1 (d) d
 0 5 −7 
92. The matrix  −5 0 11  is known as :

 7 −11 0 
(a) Upper triangular matrix (b) Skew symmetric matrix
(c) Symmetric matrix (d) Diagonal matrix
93. If A be square matrix of order n and if A = D and adj A = D′ , then :
(a) DD′ = D 2 (b) DD′ = D n −1 (c) DD′ = D n (d) none of these
94. The number of solution of the following equations x2 − x3 = 1, − x1 + 2 x3 = −2, x1 − 2 x2 = 3 is :
(a) zero (b) one (c) two (d) infinite
1 2
95. If M =  and M 2 − λ M − I 2 = 0 , then λ =
2 3

(a) −2 (b) 2 (c) −4 (d) 4


 2 5 −7 
96. The matrix  0 3 11  is known as :
 0 0 9 
(a) Symmetric matrix (b) Diagonal matrix
(c) Upper triangular matrix (d) Skew symmetric matrix
0 2   0 3a 
97. If A =  and kA =   , then the values of k , a, b are respectively :
 3 −4   2b 24 

(a) −6, − 12, − 18 (b) −6, 4, 9 (c) −6, − 4, − 9 (d) −6, 12, 18
 1 −1 
98. If A =   , then adj A is equal to :
2 3 
 −3 −1   3 1  3 −2   3 −1 
(a)  (b)  (c) (d) 
 2 −1   −2 1 
1 1 
 −2 1 
  
 

99. The inverse of a matrix A =   is :


a b
c d 
1 1 1 0
(a)  (b) (c) (d) 
 d −b   d −b  b −a 
( ad − bc )  −c a  A 0 1
   
 −c a  d −c 
 cos θ − sin θ 
100. If A =   , then which of the following statements is not correct :
 sin θ cos θ 
(a) A is orthogonal matrix (b) A′ is orthogonal matrix
(c) Determinant A = 1 (d) A is not invertible
 2 2  0 −1 
101. If A =  and B =  , then ( B −1 A−1 ) =
−1

 −3 2  1 0 
 

 2 −2   3 −2  1  2 2 1  3 2
(a)  (b)  (c) (d)
2 3  2 2  10  −2 3  10  −2 2 
   

102. The product of matrix and its transpose is an identity matrix. The value of determinant of this matrix is
(a) −1 (b) 0 (c) ±1 (d) 1

Office.: 606 , 6 Floor, Hariom Tower, Circular Road, Ranchi-1, Ph.: 0651-2562523, 9835508812, 8507613968
th
10 ( ) BY R. K. MALIK’S NEWTON CLASSES
 1 −1  a 1 
103. If A =  , B= and ( A + B ) = A2 + B 2 , then the value of a and b are :
2

 2 −1   b −1 
 

(a) a = 4, b = 1 (b) a = 1, b = 4 (c) a = 0, b = 4 (d) a = 2, b = 4


 0 −1 
104. If matrix A =  , then A16 =
1 0 

 0 −1  0 1  −1 0  1 0
(a)  (b)  (c)  (d) 
1 0  1 0  0 1 0 1
   

 1 0 −k 
105. Matrix A =  2 1 3  is invertible for :
 k 0 1 
(a) k = 1 (b) k = −1 (c) k = 0 (d) All real k
3 2 4 
1
106. If matrix A =  1 2 −1  and A−1 = adj ( A ) , then k is :
 0 1 1 
k

1
(a) 7 (b) −7 (c) (d) 11
7
0 3
107. A =  and A−1 = λ ( adj ( A ) ) , then λ =
2 0

−1 1 −1 1
(a) (b) (c) (d)
6 3 3 6
 10 0 
108. For any 2 × 2 matrix, if A ( adj A ) =   then A is equal :
 0 10 
(a) 0 (b) 10 (c) 20 (d) 100
2 3 1 4 
109. The rank of matrix, A =  0 1 2 −1  is :
 0 −2 −4 2 
(a) 2 (b) 3 (c) 1 (d) Indeterminate
 1 2 2 
1
110. The matrix A =  2 1 −2  is :
3
 −2 2 1 
(a) Orthogonal (b) Involutory (c) Idempotent (d) Nilpotent
111. Assuming that the sums and products given below are definite, which of the following is not true for
matrices :
(a) A + B = B + A (b) AB = AC does not imply B = C
(c) AB = O implies A = O or B = O (d) ( AB )′ = B′A′
 1 −1 1   4 2 2
112. Let A =  2 1 −3  and (10 ) B =  −5 0 α  . If B is the inverse of matrix A , then α is :
   
1 1 1   1 −2 3 
   
(a) 5 (b) −1 (c) 2 (d) −2

Office.: 606 , 6 Floor, Hariom Tower, Circular Road, Ranchi-1, Ph.: 0651-2562523, 9835508812, 8507613968
th
( ) BY R. K. MALIK’S NEWTON CLASSES 11

4 1 0 0
113. Rank of matrix  3 0 1 0  is :
 6 0 2 0 
(a) 4 (b) 3 (c) 2 (d) 1
 1 −1 1 
adj B
114. If A =  0 2 −3  and B = ( adj A ) , and C = 5 A , then =
 2 1 0 
C

(a) 5 (b) 25 (c) −1 (d) 1


115. If X is a square matrix of order 3 × 3 and λ is a scalar, then adj ( λ X ) is equal to :
(a) λ adj X (b) λ 3 adj X (c) λ 2 adj X (d) λ 4 adj X

116. The identify element in the group M =  , x ∈ R − {0} with respect to matrix multiplication
 x x  

is :
 x x  

1 1 11 1 1 0 0 1
(a)  (b)  (c) (d) 
1 1 21 1 0 1 1 0
    

117. The number of solutions of equations x + y − z = 0, 3 x − y − z = 0, x − 3 y + z = 0 is :


(a) 0 (b) 1 (c) 2 (d) infinite
118. In a skew symmetric matrix, the diagonal elements are all :
(a) Different from each other (b)Zero (c) One (d) none of these
2
119. If A =  2  and A = O, then the minimum value of n is :
 ab b  n

 − a − ab 
(a) 2 (b) 3 (c) 4 (d) 5
120. If AB = C , then matrices A, B, C are :
(a) A2×3 , B3×2 , C2×3 (b) A3×2 , B2×3 , C3×2 (c) A3×3 , B2×3 , C3×3 (d) A3×2 , B2×3 , C3×3
1 0 0
0 
2 3 0 0 
121. If A =  , then A is :
4 5 6 0 
7 8 9 10 
 

(a) An upper triangular matrix (b) A null matrix


(c) A lower triangular matrix (d) none of these
122. Square matrix  aij  will be an upper triangular matrix, if :
(a) aij ≠ 0, for i > j (b) aij = 0, for i > j (c) aij = 0, for i < j (d) none of these
n× n

1 2 a 0
123. Let A =   and B =   , a, b ∈ N . Then :
3 4 0 b
(a) There cannot exit any B such that AB = BA
(b) There exists more than one but finite number of B 's such that AB = BA
(c) There exists exactly one B such that AB = BA
(d) There exits infinitely many B 's such that AB = BA
124. Which of the following relations is incorrect?
(a) ( A + B + ..... + l )′ = A′ + B′ + ..... + l ′ (b) ( AB.....l )′ = A′B′......l ′

Office.: 606 , 6 Floor, Hariom Tower, Circular Road, Ranchi-1, Ph.: 0651-2562523, 9835508812, 8507613968
th
12 ( ) BY R. K. MALIK’S NEWTON CLASSES
(c) ( kA )′ = kA′ (d) ( A′ )′ = A
125. If A = diag ( 2, − 1, 3) , B = diag ( −1, 3, 2 ) , then A2 B =
(a) diag ( 5, 4, 11) (b) diag ( −4, 3, 18) (c) diag ( 3, 1, 8 ) (d) B

126. If A = [ a b ] , B = [ −b − a ] and C =   , then the correct statement is :


 a
 −a 
(a) A = − B (b) A + B = A − B (c) AC = BC (d) CA = CB
127. Which of the following is incorrect?
(a) A2 − B 2 = ( A + B )( A − B ) (b) ( AT ) = A
T

(c) ( AB ) = An B n , where A, B commutative (d) ( A − I )( I + A) = O ⇔ A2 = I


n

 a h g  x 
128. The order of [ x y z ]  h b f   y  is :
 g f c   z 
(a) 3 × 1 (b) 1× 1 (c) 1× 3 (d) 3 × 3
129. If A and B are two matrices such that A + B and AB are both defined, then :
(a) A and B are two matrices not necessarily of same order
(b) A and B are square matrices of same order
(c) Number of columns of A = number of rows of B (d) none of these
0 1
130. If A =  , I is the unit matrix of order 2 and a, b are arbitrary constants, then ( aI + bA ) is
2

0 0

equal to :
(a) a 2 I + abA (b) a 2 I + 2abA (c) a 2 I + b 2 A (d) none of these
131. Choose the correct answer
(a) Every identity matrix is a scalar matrix
(b) Every scalar matrix is an identity matrix (c) Every diagonal matrix is an identity matrix
(d) A square matrix whose each element is 1 is an identity matrix
 1 1
132. If the matrix  is commutative with the matrix  0 1  , then :
a b 

c d   
(a) a = 0, b = c (b) b = 0, c = d (c) c = 0, d = a (d) d = 0, a = b
 1 
133.  −1  [ 2 1 −1 ] =
 2 
 2   2 1 −1 
(a) [ −1] (b)  −1  (c)  −2 −1 1  (d) none of these
 −2   4 2 −2 
 1 −2   3 6 
134. If A =  and f ( t ) = t 2 − 3t + 7, then f ( A ) +   is equal to :
4 5   −12 −9 

1 0 0 0 1 0 1 1
(a)  (b)  (c)  (d) 
0 1 0 0 0 1 0 0
   

Office.: 606 , 6 Floor, Hariom Tower, Circular Road, Ranchi-1, Ph.: 0651-2562523, 9835508812, 8507613968
th
( ) BY R. K. MALIK’S NEWTON CLASSES 13

1 1 0
134. For the matrix A =  1 2 1  , which of the following is correct :
 2 1 0 
(a) A3 + 3 A2 − I = O (b) A3 − 3 A2 − I = O (c) A3 + 2 A2 − I = O (d) A3 − A2 + I = O
 1 −1  a 1 
135. If A =  , B= and ( A + B ) = A2 + B 2 , then the values of a and b are :
2

 2 −1   b −1 
 

(a) a = 4, b = 1 (b) a = 1, b = 4 (c) a = 0, b = 4 (d) a = 2, b = 4


136. If A, B are square matrices of order 3, A is non-singular and AB = O, then B is a :
(a) Null matrix (b) Singular matrix (c) Unit matrix (d) Non-singular matrix
137. If A and B are symmetric matrices of the same order, then AB − BA will be a :
(a) Symmetric matrix (b) Skew symmetric matrix
(c) Null matrix (d) None of these
138. If A and B are square matrices of the same order and AB = 3I , then A−1 is equal to :
1 1 −1
(a) 3B (b) (c) 3B −1 (d)
3 3
B B

139. If A and B are square matrices of the same order such that ( A + B )( A − B ) = A2 − B 2 , then
−1 2
( ABA ) =
(a) A2 B 2 (b) A2 (c) B 2 (d) I
140. If A is a square matrix, then which of the following matrices is not symmetric :
(a) A + A′ (b) AA′ (c) A′A (d) A − A′
141. If a matrix A is such that 3 A3 + 2 A2 + 5 A + I = 0, then its inverse is :
(a) − ( 3 A2 + 2 A + 5 I ) (b) 3 A2 + 2 A + 5 I (c) 3 A2 − 2 A − 5 I (d) none of these
142. If A is a square matrix for which aij = i 2 − j 2 , then A is :
(a) Zero matrix (b) Unit matrix (c) Symmetric matrix (d) Skew symmetric matrix
143. If I 3 is the identity matrix of order 3, then I −1
3 is :
(a) 0 (b) 3I 3 (c) I 3 (d) Does not exist
 2 2  0 −1 
144. If A =  and B =  , then ( B −1 A−1 ) =
−1

 −3 2  1 0 
 

 2 −2   3 −2  1  2 2 1  3 2
(a)  (b) (c) (d)
2 3 
 2 2  10  −2 3  10  −2 2 
 
1 2 3 
145. If A =  1 4 9  , then the value of adj A is :
 1 8 27 
(a) 36 (b) 72 (c) 144 (d) none of these
146. If A =  aij  is a square matrix of even order such that aij = i 2 − j 2 , then :
(a) A is a skew-symmetric matrix and A = 0 (b) A is symmetric matrix and A is a square
(c) A is symmetric matrix and A = 0 (d) none of these
147. Let A be an invertible matrix. Which of the following is not true?
(a) ( AT ) = ( A−1 ) (b) A−1 = A (c) (d) A−1 = A
−1 −1 2 −1 2 −1
(A ) = ( A−1 )
T

Office.: 606 , 6 Floor, Hariom Tower, Circular Road, Ranchi-1, Ph.: 0651-2562523, 9835508812, 8507613968
th
14 ( ) BY R. K. MALIK’S NEWTON CLASSES
148. If the matrix AB is zero, then :
(a) It is not necessary that either A = O or B = O (b) A = O or B = O
(c) A = O and B = O (d) all the above statements are wrong
a 0 0
149. If A =  0 a 0  , then the value of adj A , is :
 0 0 a 
(a) a 27 (b) a 9 (c) a 6 (d) a 2
150. If B is a non-singular matrix and A is a square matrix, then det ( B −1 AB ) is equal to :

(a) det ( A−1 ) (b) det ( B −1 ) (c) det ( A) (d) det ( B )


151. If A, B are square matrices of order 3, A is non-singular and AB = O, then B is a :
(a) null matrix (b) singular matrix (c) unit matrix (d) non-singular matrix
1 − 2i 
152. The matrix A =  is which of the following?
 i
 −1 − 2i 0 
(a) symmetric (b) skew-symmetric (c) hermitian (d) skew-hermitian
α 0  1 0
153. If A =  and B =  , then the value of α for which A2 = B, is :
 1 1 5 1
 

(a) 1 (b) –1 (c) 4 (d) none of these


 1 2 −3 
154. The element in the first row and third column of the inverse of the matrix  0 1 2  , is :
 0 0 1 
(a) –2 (b) 0 (c) 1 (d) none of these
 a b3   1 8 
−1

155. If  , then 
a b
2 0  2 0 2 0 
=  =

 0 −2  1 0  0 −8   0 1/ 2 
(a)  (b) (c)  (d) 
 −2 1 
0 1   −2 1   1/ 2 −1/ 4 
  

156. If A is a square matrix such that A2 − A + I = 0, the inverse of A is :
(a) I − A (b) A − I (c) A (d) A + I
157. If A is 3 × 3 matrix and B is its adjoint such that B = 64, then A =
(a) 64 (b) ±64 (c) ±8 (d) 18
1 2 2 
1
158. If A =  2 1 −2  is an orthogonal matrix, then :
3
 a 2 b 
(a) a = 2, b = 1 (b) a = −2, b = −1 (c) a = 2, b = −1 (d) a = −2, b = 1
ω 0 
159. If ω is a complex cube root of unity and A =  , then A100 is equal to :
0 ω

(a) A (b) − A (c) O (d) none of these


 0 1 −1 
160. If A =  2 1 3  , then ( A ( adj A ) A−1 ) A =
 3 2 1 

Office.: 606 , 6 Floor, Hariom Tower, Circular Road, Ranchi-1, Ph.: 0651-2562523, 9835508812, 8507613968
th
( ) BY R. K. MALIK’S NEWTON CLASSES 15

3 0 0  −6 0 0   0 1/ 6 −1/ 6 
(a) 2  0 3 0  (b)  0 −6 0  (c)  2 / 6 1/ 6 3 / 6  (d) none of these
 0 0 3   0 0 −6   3 / 6 2 / 6 1/ 6 
161. If A is a non-singular matrix of order 3 × 3, then adj ( adj A ) is equal to :

(a) A A (b) A A (c) A (d) none of these


2 −1
A
162. If A and B are square matrices of order 3 × 3 such that A is an orthogonal matrix and B is a skew-
symmetric matrix, then which of the following statement is true?
(a) AB = 1 (b) AB = 0 (c) AB = −1 (d) none of these
 0 0 −1 
163. Let A =  0 −1 0  . The only correct statement about the matrix A is :
 −1 0 0 
(a) A−1 does not exist (b) A = ( −1) I is a unit matrix
(c) A is a zero matrix (d) A2 = I
 1 −1 1   4 2 2
164. Let A =  2 1 −3  and 10 B =  −5 0 α  . If B is inverse of matrix A, then α is :
 
 1 1 1   1 −2 3 
(a) 2 (b) –1 (c) 3 (d) 5
165. If A is a square matrix such that A2 = A and B = I − A, then AB + BA + I − ( I − A ) =
2

(a) A (b) 2 A (c) − A (d) I − A


3 2 4
166. If A =  1 2 1  and Aij are the cofactor of aij , then a11 A11 + a12 A12 + a13 A13 is equal to :
 3 2 6 
(a) 8 (b) 6 (c) 4 (d) 0
3 5  1 17 
167. If A =  and B =   , then AB is equal to :
2 0  0 −10 

(a) 80 (b) 100 (c) –110 (d) 92

168. If Ai =  i  and if a < 1, b < 1 then ∑ det ( A ) is equal to :


 ai bi  ∞

i =1
i
b ai 
a2 b2 a 2 − b2 a2 b2 a2 b2
(a) (b) (c) (c)
(1 − a )
2
(1 − b )
2
(1 − a 2 )(1 − b2 ) (1 − a )
1
(1 − b )
2
(1 + a )
2
(1 + b )
2
− + −

169. Let the determinant of a 3 × 3 matrix A be 6, then B is a matrix defined by B = 5 A2 . Then,


determinant of B is :
(a) 180 (b) 100 (c) 80 (d) none of these
2 1 0
170. If A =  0 2 1  , then adj A is equal to :
 1 0 2 
1
(a) 0 (b) 9 (c) (d) 81
9
171. If A and B are square matrices of the same order and AB = 3I , then A−1 is equal to :

Office.: 606 , 6 Floor, Hariom Tower, Circular Road, Ranchi-1, Ph.: 0651-2562523, 9835508812, 8507613968
th
16 ( ) BY R. K. MALIK’S NEWTON CLASSES
1 1 −1
(a) 3B (b) (c) 3B −1 (d)
3 3
B B

 1 2
172. Let A =  and A−1 = xA + yI , then the value of x and y are :
 −5 1 

−1 2 −1 −2 1 2 1 −2
(a) x = , y = (b) x = , y = (c) x = , y = (d) x = , y=
11 11 11 11 11 11 11 11
 1 −1 1 
adj B
173. If A =  0 2 −3  and B = ( adj A ) , and C = 5 A, then is equal to :
 2 1 0 
C

(a) 5 (b) 25 (c) –1 (d) 1


 1 0 2   5 a −2 
174. adj  −1 1 −2  =  1 1 0  , then [ a b ] is equal to :
 0 2 1   −2 −2 b 
(a) [ −4 1] (b) [ −4 − 1] (c) [ 4 1] (d) [ 4 − 1]
175. If k is a scalar and I is a unit matrix of order 3, then adj ( kI ) is equal to :
(a) k 3 I (b) k 2 I (c) − k 3 I (d) − k 2 I
 −1 2 5 
176. If the rank of the matrix  2 −4 a − 4  is 1, then the value of a is :

 1 −2 a + 1 
(a) –1 (b) 2 (c) –6 (d) 4
 −3   4   10 
177. If x   + y   =   , then :
 4   3   −5 
(a) x = −2, y = 1 (b) x = −9, y = 10 (c) x = 22, y = 1 (d) x = 2, y = −1
4 1 0 0
178. The rank of the matrix  3 0 1 0  is :
 6 0 2 0 
(a) 4 (b) 3 (c) 2 (d) 1
 2 4 5 
179. If A =  4 8 10  , the rank of A is equal to :
 −6 −12 −15 
(a) 0 (b) 1 (c) 2 (d) 3
 1 −1 1 
180. The rank of  1 1 −1  is :
 −1 1 1 
(a) 0 (b) 1 (c) 2 (d) 3
181. If A is a non-singular matrix such that A = A + I , then the inverse of B = A6 − A5 is
3

(a) − A−1 (b) − A (c) A (d) A−1


182. If A and B are square matrices of order 3 which satisfy AB = A and BA = B , then ( A + B ) is equal
7

to
(a) 7 ( A + B ) (b) 72 ( A + B ) (c) 8 ( A + B ) (d) 82 ( A + B )

Office.: 606 , 6 Floor, Hariom Tower, Circular Road, Ranchi-1, Ph.: 0651-2562523, 9835508812, 8507613968
th
( ) BY R. K. MALIK’S NEWTON CLASSES 17
1 2 
183. Let A =  . Then, An is equal to
0 1 

1 n   n 2n   1 2n  n 2
(a)  (b)  (c)  (d) 
0 1  0 n  0 1  0 n 
   

184. If A and B are two square matrices of the same order and m is a positive integer, then
( A + B) = mC0 Am + mC1 Am −1 B + mC2 Am − 2 B 2 + ... + mCm B m , if
m

(a) AB = − BA (b) AB = 2 BA (c) AB = AB (d) Am = O, B m = O

185. If A =   is such that A = 0 and A − ( a + b ) A + kI = O , then k is equal to


a b  2

c d
(a) b + c (b) a + d (c) zero (d) ab + cd
0 1 
186. If A =  , I is the unit matrix of order 2 and a, b are arbitary constants, then ( al + bA ) is equal
2

0 0 

to
(a) a 2 I + abA (b) a 2 I + 2abA (c) a 2 I + b 2 A (d) None of these
a b c 
187. If matrix A =  b c a  , where a, b, c are real positive numbers, abc = 1 and AT A = I , then find the
 c a b 
value of a 3 + b3 + c 3 .
(a) 0 (b) 1 (c) 3 (d) 4
1 −1 a 1 
188. If A =  , B= and ( A + B ) = A2 + B 2 , then the values of a and b are
2

 2 −1  b −1
 

(a) a = 0, b = 4 (b) a = 1, b = 4 (c) a = 2, b = 4 (d) a = 4, b = 2


189. Let A and B be two symmetric matrices of same order. Then, the matrix ( AB − BA) is
(a) a null matrix (b) the identity matrix
(c) a symmetric matrix (d) a skew-symmetric matrix
1 2 2
1
190. The matrix A =  2 1 −2  is which of the following?
3
 −2 2 −1
(a) Orthogonal (b) Involutary (c) Idempotent (d) Nilpotent
b2 
191. If A =  2  , then A is a
 ab
 − a − ab 
(a) scalar matrix (b) nilpotent matrix (c) idempotent matrix (d) involuntary matrix
cos θ − sin θ 
192. If A =   , then which of the following statements is not correct?
 sin θ cos θ 
(a) A is orthogonal matrix (b) A′ is orthogonal matrix
(c) determinant A = 1 (d) A is not invertible
193. If A and B are Hermitian matrices of same order, then ( AB − BA) is
(a) a Null matrix (b) a Harmitian matrix (c) a Skew-Hermitian matrix (d) a Diagonal matrix
194. If A is orthogonal, then

Office.: 606 , 6 Floor, Hariom Tower, Circular Road, Ranchi-1, Ph.: 0651-2562523, 9835508812, 8507613968
th
18 ( ) BY R. K. MALIK’S NEWTON CLASSES
1
(a) A = 0 (b) A = ±1 (c) A = ±2 (d) A = ±
2
195. Let the determinant of a 3 × 3 matrix A be 6 and B is a matrix defined by B = 5 A2 . Then, det of B is
(a) 80 (b) 100 (c) 180 (d) None of these
196. If a square matrix A is such that AAT = I = AT A , then A is equal to
1
(a) 0 (b) ± 1 (c) ± 2 (d) ±
2
r − 1
197. If the matrix is given by , r = 1, 2, 3, ... , then the value of
 r
r − 1 r 
Mr Mr = 

det ( M 1 ) + det ( M 2 ) + ... + det ( M 2008 ) is

(a) 2007 (b) ( 2007 ) (c) 2008 (d) ( 2008 )


2 2

198. A determinant of second order is made with elements 0 and 1 only. What is the probability that the
determinant is non-negative?
7 11 13 15
(a) (b) (c) (d)
12 12 16 16
199. If A be any square matrix of order n and if A = D and adj A = D′ , then
(a) DD′ = D 2 (b) DD′ = D n −1 (c) DD′ = D n (d) None of these
200. If A ( adj A ) = 5I where I is the identity matrix of order 3, then adj A is equal to
(a) 5 (b) 10 (c) 25 (d) 125
201. Let A be a square matrix of order 3. If A = −2 , then the value of the determinant of A adj A . is
equal to
(a) −32 (b) −8 (c) 8 (d) 32
202. If A and B are square matrices of the same order such that ( A + B )( A − B ) = A2 − B 2 , then ( ABA−1 )
2

is equal to
(a) I (b) A2 (c) B 2 (d) A2 B 2
203. If A is a 3 × 3 non-singular matrix such that det ( A) = ∆ , then det ( adj A−1 ) equals
(a) ∆ −1 (b) ∆ −2 (b) ∆ 2 (d) ∆ 3
204. If A is a square matrix such that ( A − 2 I )( A + I ) = O , then A−1 is equal to

(a) 2A − I (b) 2A + I (c) (d)


A− I A+ I
2 2
205. For the system of equations a1 x + b1 y + c1 z = 0, a2 x = b2 y + c2 z = 0 and a3 x + b3 y + c3 z = 0 , if
a1 b1 c1
a2 b2 c2 = 0 , then the system has
a3 b3 c3
(a) no solution (b) only trivial solution ( 0, 0, 0 )
(c) one trivial and only one non-trivial solution (d) more than two solution
206. If the system of equations 2 x − y − z = 2, x − 2 y + z = −4 and x + y + λ z = 4 does not have unique
solution, then λ is equal to
(a) −3 (b) −2 (c) 0 (d) 3

Office.: 606 , 6 Floor, Hariom Tower, Circular Road, Ranchi-1, Ph.: 0651-2562523, 9835508812, 8507613968
th
( ) BY R. K. MALIK’S NEWTON CLASSES 19
207. If the system of equations x + 2 y − z = 1, ( λ + 3) z = 3 , ( 2λ + 1) x + z = 0 is does not have unique
solution then the value of λ is
1
(a) −3 (b) 0 (c) (d) 2
2
208. If the trivial solution is the only solution of the system of equations x − ky + z = 0 , kx + 3 y − kz = 0 ,
3 x + y − z = 0 , then the set of all values of k is
(a) R − {2} (b) R − {2, − 3} (c) {2, − 3} (d) R − {−3}
209. If the system of equations x + ay = 0, az + y = 0 and ax + z = 0 has infinite number of solutions, then
the value of a is
(a) −1 (b) 0 (c) 1 (d) No real values
210. For what value of λ , the system of equations x + y + z = 6, x + 2 y + 3 z = 10, x + λ z = 12 does not have
unique solution
(a) λ = −2 (b) λ = 1 (c) λ = 2 (d) λ = 3
211. If the system of equations x − ky − z = 0, kx − y − z = 0, x + y − z = 0 has a non-zero solution, then the
possible values of k are:
(a) −1, 1 (b) −1, 2 (c) 1, 2 (d) 0, 1
212. If the equations x + ay − z = 0 , 2 x − y + az = 0 and ax + y + 2 z = 0 will have non- trivial solution if a
is equal to
(a) 2, 3 (b) −2, 2 (c) 2 ± 3, − 2 (d) 1 ± 3, − 2
213. If x, y, z are not all zero and x + 2 y = bz , y + 2 x = az , bx + ay = z , then a 2 + b 2 is equal to
(a) 4ab − 3 (b) 4ab − 1 (c) 4ab + 1 (d) 4ab + 3
214. The set of equations a + b − 2c = 0, 2a − 3b + c = 0 and a − 5b + 4c = α is consistent for α equal to
(a) −1 (b) 0 (c) 1 (d) 2
215. If the three linear equations x + 4 zy + az = 0, x + 3by + bz = 0 and x + 2cy + cz = 0 have a non-trival
solution then a, b, c are in
(a) A.P. (b) G.P. (c) H.P. (d) None of these
216. If the system of linear equations x + 2 y − 3 z = 1, ( p + 2 ) z = 3 , ( 2 p + 1) y + z = 2 has no solution, if
1
(a) p = −2 (b) p = (c) p = 2 (d) p = 3
2
217. The system of equations : x + y + z = 6, x + 2 y + 3 z = 10 , x + 2 y + λ z = µ , has no solution for
(a) λ ≠ 3, µ ≠ 10 (b) λ ≠ 3, µ = 10 (c) λ = 3, µ ≠ 10 (d) None of these
218. if the equations x = cy + bz , y = az + cz and z = bx + ay have a solution other than x = 0, y = 0, z = 0 ,
then a, b, c are connected by the relation
(a) a 2 + b 2 + c 2 − bc − ca − ab = 0 (b) a 3 + b3 + c3 + 3abc = 0
(c) a 2 + b 2 + c 2 + 2abc = 1 (d) a 2 + b 2 + c 2 − ( a + b + c ) = 1
1 1 0 
219. The rank of the matrix 1 1 0  is
1 1 0 
 

(a) 0 (b) 1 (c) 2 (d) 3

Office.: 606 , 6 Floor, Hariom Tower, Circular Road, Ranchi-1, Ph.: 0651-2562523, 9835508812, 8507613968
th
20 ( ) BY R. K. MALIK’S NEWTON CLASSES
 1 −1 1 
220. The rank of  1 1 −1 is
 −1 1 1 
 

(a) 0 (b) 1 (c) 2 (d) 3


 1 2 −1
221. If A =  −1 1 2  , then ρ ( A) is equal to
 2 −1 1 
(a) 0 (b) 1 (c) 2 (d) 3
2 4 5 
222. If A =  4
 8 10  , then rank of A is equal to
 −6 −12 −15
(a) 0 (b) 1 (c) 2 (d) 3
 −1 2 5 
223. The rank of the matrix  2 −4 a − 4  is

 1 −2 a + 1 
(a) 1 if a = 6 (b) 2 if a = 1 (c) 3 if a = 2 (d) 4 if a = −6
224. The values of α for which the system of equations :
x + y + z = 1, x + 2 y + 4 z = α , x + 4 y + 10 z = α 2 is consistent, are given by
(a) 1, − 2 (b) 1, 2 (c) 2 (d) 1
3 4 5 6
7
4 5 6 7 8 
225. The rank of the matrix  is equal to
5 6 7 8 9
10 11 12 13 14 
 

(a) 1 (b) 2 (c) 3 (d) 4


1 1 0
226. For the matrix A = 1 2 1  , which is correct?
 2 1 0
(a) A3 − A2 + I = O (b) A3 + 3 A2 − I = O (c) A3 − 3 A2 − I = O (d) A3 + 2 A2 − I = O
 −3 4 
227. If A =   , then its characteristic roots are
 2 −1
(a) 1, − 5 (b) 1,5 (c) −1, 5 (d) −1, − 5
5 4 
228. The eigen values of the matrix A =   are
1 2 
(a) 3, − 3 (b) 1, − 2 (c) 1, 6 (d) 2, 4
1 0 0
229. The characteristic roots of the matrix  2 3 0 are
 4 5 6
(a) 1, 2, 4 (b) 1, 3, 6 (c) 2, 4, 6 (d) 4, 5, 6

Office.: 606 , 6 Floor, Hariom Tower, Circular Road, Ranchi-1, Ph.: 0651-2562523, 9835508812, 8507613968
th
( ) BY R. K. MALIK’S NEWTON CLASSES 21
 1 3
230. If A =   , then A equals
−1

 4 5
1 1 1
(a) ( A − 6I ) (b) ( A + 6I ) (c) ( 6 I − A) (d) None of these
7 7 7
1 1 0 
231. If A = 0 1 1  , then A−1 is equal to
0 0 1 

(a) A2 − 2 I 3 (b) A2 − 2 A + 2 I 3 (c) A2 + 3 A − 2 I 3 (d) A2 − 3 A + 3I 3


1 −1 a 1 
232. If A =  , B= and ( A + B ) = A2 + B 2 , then the values of a and b are
2

 2 −1  b −1
 

(a) a = 4, b = 1 (b) a = 1, b = 4 (c) a = 0, b = 4 (d) a = 2, b = 4


233. If A, B are two n × n non-singular matrices, then
(a) AB is non-singular (b) AB is singular (c) ( AB ) = A−1 B −1 (d) ( AB ) does not exist
−1 −1

234. If A, B are square matrices of order 3, A is non-singular and AB = O, then B is a


(a) null matrix (b) singular matrix (c) unit matrix (d) non-singular
matrix.
1 − 2i 
235. The matrix A =  is which of the following ?
 i
 −1 − 2i 0 
(a) symmetric (b) skew-symmetric (c) hermitian (d) skew-hermitian
1 2 −3
236. The element in the first row and third column of the inverse of the matrix 0 1 2  , is
0 0 1 
(a) −2 (b) 0 (c) 7 (d) none of these
1 2 2 
237. If A =  2 1 −2 is an orthogonal matrix, then
1
3
 a 2 b 
(a) a = 2, b = 1 (b) a = −2, b = −1 (c) a = 2, b = −1 (d) a = −2, b = 1
 0 1 −1
238. If A =  2 1 3  , then ( A ( adj A ) A−1 ) A =
 3 2 1 

3 0 0  −6 0 0   0 1/ 6 −1/ 6 
(a) 2 0 3 0  (b)  0 −6 0  (c)  2 / 6 1/ 6 3 / 6  (d) none of these
0 0 3  0 0 −6   3 / 6 2 / 6 1/ 6 
239. If A and B are square matrices of order 3 × 3 such that A is an orthogonal matrix and B is a
skew-symmetric matrix, then which of the following statements is true ?
(a) AB = 1 (b) AB = 0 (c) AB = −1 (d) none of these
 1 −1 1   4 2 2
240. Let A =  2 1 −3 and 10 B =  −5 0 α  if B is the inverse of matrix A, then α is
 
1 1 1   1 −2 3 

Office.: 606 , 6 Floor, Hariom Tower, Circular Road, Ranchi-1, Ph.: 0651-2562523, 9835508812, 8507613968
th
22 ( ) BY R. K. MALIK’S NEWTON CLASSES
(a) 2 (b) −1 (c) 3 (d) 5
241. If A is a square matrix such that A2 = A and B = I − A, then AB + BA + I − ( I − A ) =
2

(a) A (b) 2 A (c) − A (d) I − A


242. Statement-1: If A is an orthogonal matrix of order n, then adj ( adj A ) = ±1
n2 −1
Statement-2: adj ( adj A ) = A
(a) 1 (b) 2 (c) 3 (d) 4
243. If A is an orthogonal matrix, then A equals −1

(a) A (b) AT (c) A2 (d) none of these


244. If A, B are symmetric matrices of the same order then AB − BA is
(a) symmetric matrix (b) skew-symmetric matrix
(c) null matrix (d) unit matrix
245. If A is a singular matrix, then A ( adj A) is
(a) identity matrix (b) null matrix (c) scalar matrix (d) none of these
 cos x sin x  1 0 
246. If A =  and A ( adj A ) = k   , then the value of k is
 − sin x cos x  0 1 

(a) sin x cos x (b) 1 (c) 2 (d) 3


247. If A =  aij  is a square matrix of order n × n and k is a scalar, then kA =

(a) k n A (b) k A (c) k n −1 A (d) none of these

 1 − tan θ   1 tan θ 
−1

248. If   , then
 a −b 
 tan θ 1   − tan θ 1 
   =
b a 
(a) a = 1, b = 1 (b) a = cos 2θ , b = sin 2θ (c) a = sin 2θ , b = cos 2θ (d) none of these
249. If a matrix A is such that 3 A3 + 2 A2 + 5 A + I = 0, then A−1 is equal to
(a) − ( 3 A2 + 2 A + 5 ) (b) 3 A2 + 2 A + 5 (c) 3 A2 − 2 A − 5 (d) none of these
250. If A is an invertible matrix, then which the following is correct
(a) A−1 is multi valued (b) A−1 is singular (c) ( A−1 ) ≠ ( AT ) (d) A ≠ 0
T −1

251. If for a matrix A, A2 + I = O, where I is the identity matrix, then A equals


1 0   −i 0   1 2  −1 0 
(a)  (b)  (c)  (d) 
0 1   0 −i   −1 1   0 −1
   

252. Let A be a skew-symmetric matrix of odd order, then A is equal to


(a) 0 (b) 1 (c) −1 (d) none of these
253. If A is an orthogonal matrix, then
(a) A = 0 (b) A = ±1 (c) A = ± 2 (d) none of these
254. If A is a non-singular square matrix of order n, then the rank of A is
(a) equal to n (b) less than n (c) greater than n (d) none of these
255. Let A =  aij  be a matrix such that aij = 1 for all i, j. Then,

(a) rank ( A ) > 1 (b) rank ( A ) = 1 (c) rank ( A ) = m (d) rank ( A ) = n


m× n

Office.: 606 , 6 Floor, Hariom Tower, Circular Road, Ranchi-1, Ph.: 0651-2562523, 9835508812, 8507613968
th
( ) BY R. K. MALIK’S NEWTON CLASSES 23
1 2 3 0
2 4 3 2 
256. The rank of the matrix  is
3 2 1 3
6 8 7 5
 

(a) 1 (b) 2 (c) 3 (d) 4


2 3 1 4 
257. The rank of the matrix A =  0 1 2 − 1  is
 0 −2 −4 2 
(a) 2 (b) 3 (c) 1 (d) indeterminate
258. Let a, b, c be positive real numbers. The following system of equations in x, y and z
x2 y2 z2 x2 y 2 z 2 x2 y 2 z 2
1, 1, + = 1 has
a2 b2 c2 a 2 b2 c2 a 2 b2 c2
+ − = − + = − +

(a) no solution (b) unique solution


(c) infinitely many solutions (d) finitely many solutions
259. If A and B are square matrices of order 3 such that A = −1, B = 3, then 3AB equals
(a) −9 (b) −81 (c) −27 (d) 81
 x1 y1 1
260. If the points ( x1 , y1 ) , ( x2 , y2 ) and ( x3 , y3 ) are collinear, then the rank of the matrix  x2 y2 1
 x3 y3 1
will always be less than :
(a) 3 (b) 2 (c) 1 (d) none of these
 x 2 −1 
−11 
261. If A =  −1 1 2  ;  x ≠  and det ( adj ( adj A ) ) = (14 ) . Then the value of x is
4

3 

 2 −1 1 

(a) 2 (b) –2 (c) 0 (d) none of these


262. If A, B and C are matrices such that AB = AC , then
(a) B = C (b) A = 0
(c) Al least one of A and ( B − C ) is singular (d) At least one of A and ( B − C ) is non-singular
263. Which of the following statement, is correct?
(a) A skew-symmetric matrix of odd order is always invertible.
(b) A skew-symmetric matrix of odd order is always non-invertible.
(c) A skew-symmetric matrix of even order is always non-invertible. (d) none of these
 a11 a12 a13   1 2 −1 
264. If matrix  a21 a22 a23  is the inverse of matrix  0 2 1  , then the value of 10a is :
23

 0 0 5 
 
 a31 a32 a33 
(a) −1 (b) 2 (c) 1 (d) 0
265. A is an invertible matrix and the inverse of A is B . If C is inverse of B and A = 3, then the value
of C is :
1 1
(a) (b) 3 (c) (d) 1
3 9

Office.: 606 , 6 Floor, Hariom Tower, Circular Road, Ranchi-1, Ph.: 0651-2562523, 9835508812, 8507613968
th
24 ( ) BY R. K. MALIK’S NEWTON CLASSES
 −1 18 
 7 −10 17 
266. If 3 A + 4 B′ =  , 2 B − 3 A′ =  4 −6  , then B =
 0 6 31   −5 −7 

 1 −3   1 3  1 −4 
 1 −2 3 
(a)  −1 0  (b)  −1 0  (c)  (d)  −2 2 
 −4 2 5 
   
 2 4   2 4   3 5 

267. A is a third order real matrix and the square of the determinant f its cofactor matrix is 28581. Then
A =
(a) 26 (b) 13 (c) 169 (d) –169
4 0 0
adj ( adj A )
268. If A is a square matrix such that A ( adj A ) = 0 4 0 , then is equal to :
adj A
0 0 4
(a) 256 (b) 64 (c) 32 (d) 16

269.   is a square root of the unit matrix of second order if δ is equal to


α β 
γ δ 
(a) α (b) β (c) γ (d) none of these
270. If the system of equations a 2 x − ay = 1 − a and bx + ( 3 − 2b ) y = 3 + a posses a unique solution
x = 1, y = 1, then
(a) a = 1, b = −1 (b) a = −1, b = 1 (c) a = 0, b = 0 (d) None of these
271. The system of equations x + y + z = 8, x − y + 2 z = 6,3 x + 5 y − 7 z = 14 has
(a) a unique solution (b) no solution (c) infinite number of solutions (d)none of the above
272. If A and B are symmetric matrices and AB = BA, then A−1B is a :
(a) symmetric matrix (b) skew-symmetric matrix (c)unit matrix (d) None of these
273. If A and B are square matrices of the same order and AB = 3I , then A−1 is equal to :
1 1
(a) 3B (b) (c) 3B −1 (d) B −1
3 3
B

274. The system of equations : x + y + z = 6 , x + 2 y + 3 z = 10 , x + 2 y + λ z = µ has no solution for


(a) λ ≠ 3, µ = 10 (b) λ = 3, µ ≠ 10 (c) λ ≠ 3, µ ≠ 10 (d) None of these
275. If A3 = O, Then I + A + A2 equals
(a) I − A (b) ( I − A ) (c) ( I + A ) (d) None of these
−1 −1

276. The inverse of a symmetric matrix is :


(a) symmetric (b) skew-symmetric (c) diagonal matrix (d) None of these
277. If A is a symmetric matrix and n ∈ N , then A is
(a) symmetric (b) skew-symmetric (c) a diagonal matrix (d) None of these
n

278. If A is a skew-symmetric matrix and n is a positive integer, then A is


(a) a symmetric matrix (b) skew-symmetric matrix (c) diagonal matrix (d) None of these
n

279. If A and B are square matrices of order n, then A − λ I and B − λ I commute for every scalar λ , only
if :
(a) AB = BA (b) AB + BA = O (c) A = − B (d) None of these.

Office.: 606 , 6 Floor, Hariom Tower, Circular Road, Ranchi-1, Ph.: 0651-2562523, 9835508812, 8507613968
th
( ) BY R. K. MALIK’S NEWTON CLASSES 25
1 1
280. If both A − I and A + are orthogonal matrices, then
2 2
(a) A is orthogonal (b) A is skew-symmetric matrix of even order
3
(c) A2 = I (d) None of these
4
281. If A is symmetric as well as skew-symmetric matrix, then A is :
(a) diagonal matrix (b) null matrix (c) triangular matrix (d) none of these.
282. Every square matrix (with complex elements) can be uniquely expressed as the sum of a Hermitian and
Skew- Hermitian matrices.
(a) True (b) False
Paragraph for Q.283 to Q.285
Let A and B are two square matrices such that B = A−1 BA. For the matrices A and B solve each of
the following.
283. equal to :
2
( A − B)
(a) 0 (b) A2 + B 2 (c) A2 − 2 AB + B 2 (d) A − B
284. If AB = − BA also, then ( A + B ) equal to :
2

(a) 0 (b) A2 (c) A + B (d) A + B + AB + BA


285. If AB = −2 BA also, then ( A + B ) equal to :
3

(a) 0 (b) A3 + A2 + A (c) A3 (d) A2 + AB + BA + B 2

Assertion reason Type


Choose the correct option for given Statement-1 and Statement-2
(a) Statement I is true, Statement II is true; Statement II is a correct explanation for Statement I
(b) Statement I is true, Statement II is true; Statement II is not a correct explanation for Statement I
(c) Statement I is true, Statement II is false (d) Statement I is false, Statement II is true
286. Let A be a 2 × 2 matrix with non-zero entries and let A2 = I , where I is 2 × 2 identity matrix.
Define Tr ( A ) = sum of diagonal elements of A and A = determinant of matrix A.
Statement 1 : Tr ( A) = 0 Statement 2 : A = 1.
287. Statement 1 : For a singular matrix A , if AB = AC ⇒ B = C
Statement 2 : If A = 0 , then A−1 does not exist
288. Let A be a square matrix of order n .
Statement 1 : adj ( adj A ) = A Statement 2 : adj ( adj A ) = A
( n −1)2 n−2
A

289. Statement 1 : If A is an orthogonal matrix of order n, then adj ( adj A ) = ±1 .

Statement 2 : adj ( adj A ) = A .


n 2 −1

290. Let A be 2 × 2 matrix.


Statement 1 : adj ( adj A ) = A Statement 2 : adj A = A

Office.: 606 , 6 Floor, Hariom Tower, Circular Road, Ranchi-1, Ph.: 0651-2562523, 9835508812, 8507613968
th
JEE (MAIN & ADV.), MEDICAL + BOARD, NDA, X & IX
Enjoys unparalleled reputation for best results
in terms of percentage selection
www.newtonclasses.net

SOLUTION OF MATRICES ( )
Only one option is correct.
 1 * *
1. Ans. (d), Consider  * 1 *  . By placing a1 in any one of the 6* position and 0 elsewhere. We get 6
 
* * 1
non-singular matrices.
 

* * 1
Similarly,  * 1 *  gives at least one non-singular.
 
 1 * *
 
2. Ans. (b), We have, A2 − B 2 = ( A − B )( A + B ) ⇒ A2 − B 2 = A2 + AB − BA − B 2 ⇒ AB = BA
3. Ans. (b), From given matrix equation, we have
x− y−z =0
− y+z =5
z =3
⇒ x = 1, y = −2, z = 3
4. Ans. (b), Given, A = A ', B = B ' …(i)
Now, ( AB − BA) ' = ( AB ) '− ( BA) '
= B ' A '− A ' B ' ∵ AB ′ = B′A′
 ( ) 
= BA − AB [from (i)]
= − ( AB − BA )
∴ AB −BA is a skew-symmetric matrix.

5. Ans. (a), Now, A + AT( ) ( ) ∴ A + AT is symmetric matrix.


T T
= AT + AT = AT + A

x 1
6. Ans. (b), A = 
1 0

 x 1  x 1   x2 + 1 x   1 0 
∴ A2 =  ⇒ x 2 + 1 = 1, x = 0 ⇒ x=0
1 01 0 1 0 1
  ⇒  = 
 x
ω 0 
7. Ans. (b), Given, A = 
0 ω

 ω 0   ω 0   ω2 0 
A2 = 
 0 ω  0 ω   0 ω 
 = 2 

 ω2 0   ω 0   ω3 0 
3

 0 ω  0 ω   0 ω 
2  3 
A = =

 ω 50 0 
Similarly, A50 = 
 0 ω 
50 

Office.: 606 , 6 Floor, Hariom Tower, Circular Road, Ranchi-1, Ph.: 0651-2562523, 9835508812, 8507613968
th
2 ( ) BY R. K. MALIK’S NEWTON CLASSES
 ω 3 16 ω 2
( ) 0  2
0  ω 0 
 = ω
 0 ω  = ωA
0
=
0
2
= ω
3 16
 
 2 
 ( )
ω ω   ω   

8. Ans. (b), det ( 2 AB ) = 23 det ( A ) det ( B ) [∵ order of square matrix is 3]


= 8det ( A ) det ( B )
9. Ans. (c), Given, 2 X + 3Y = O …(1)
and X + 2Y = I …(2)
On solving Eqs. (1) and (2), we get
 −3 0 
X = −3 I = 
 0 −3 

 1 −2   1 −2   −7 −12 
10. Ans. (b), A2 = 
 4 5   4 5   24 17 
 =  

Now, f ( A) = A2 − 3 A + 7 I
 −7 −12   1 −2   1 0   −3 −6 
−3 +7
 24 17  4 5   0 1   12 9 
=    = 

 3 6   −3 −6   3 6  0 0
 −12 −9   12 9   −12 −9   0 0 
∴ f ( A) +  =
  +
   = 

11. Ans. (a), Since, given matrix is a triangular matrix, so its characteristic roots are the diagonal elements.
Hence, required roots are 1, 3, 6.
1 1 1
12. Ans. (d), Given, A = 3  1 1 1 
 1 1 1 

1 1 1 1 1 1 3 3 3 1 1 1
∴ A = 3  1 1 1  .3  1 1 1  = 9  3 3 3  = 9 × 3 × 1 1 1 = 9 A
2      
 1 1 1   1 1 1   3 3 3  1 1 1
∴ A4 = A2 . A2 = 9 A.9 A = 81A2 = 81× 9 A = 729 A
 1 2 3   x
13. Ans. (c), Given, [1 x 1]  0 5 1   1  = 0
 0 3 2   −2 

 x+2−6  x−4
 = 0 ⇒ 1 x 1  3  = 0 ⇒ x − 4 + 3x − 1 = 0 ⇒ x = 5

⇒ [1 x 1]  0 + 5 − 2
4
[ ]
 0 + 3 − 4  −1 
 


 0 a   0 a   0 a   ab 0 
2

14. Ans. (d), 


 b 0   b 0   b 0   0 ab 
 =  = 

 0 a   0 a   0 a   a 2b 2 0  1 0
4 2 2

and  [given]
b 0 b 0 b 0  0 a b  0 1
 =    = 2 2
= 

⇒ a 2b 2 = 1 ⇒ ab = 1
15. Ans. (b), We have,

Office.: 606 , 6 Floor, Hariom Tower, Circular Road, Ranchi-1, Ph.: 0651-2562523, 9835508812, 8507613968
th
( ) BY R. K. MALIK’S NEWTON CLASSES 3
 2 −1 4 
2 A + 3B =  …(1)
3 2 5

5 0 3 
and A + 2B =  …(2)
1 6 2 

On multiplying Eq. (2) by 2 and then subtracting Eq. (1) from Eq. (2), we get
5 0 3  2 −1 4  8 1 2
B = 2
1 6 2  3 2 5   −1 10 −1 
 −  = 

16. Ans. (d), O ( A ') = 3 × 2, O ( B ') = 2 × 3


(a) CB + A '
Now, order of CB
= (order of C is 3 × 3 ) (order of B is 3 × 2 )
= order of CB is 3 × 2
Since, O ( A ') = 3 × 2
∴ Matrix CB + A ' can be determined
(b) O ( BA) = 3 × 3 [∵ O ( B ) is 3 × 2 and O ( A) is 2 × 3 ]
and O ( C ) = 3 × 3
∴ Matrix BAC can be determined, i.e., 3 × 3
(c) O ( A + B ') = 2 × 3 [∵ O ( A) is 2 × 3 and O ( B′ ) is 2 × 3 ]
⇒ O ( A + B ') ' = 3 × 2
and O ( C ) = 3 × 3
∴ Matrix C ( A + B ') ' can be determined, i.e., 3 × 2
(d) O ( A + B ') = 2 × 3
and O ( C ) = 3 × 3
∴ Matrix C ( A + B ') cannot be determined.

1  1 1
'

17. Ans. (b),  ( A − A ' )  = ( A − A ' ) ' = ( A '− A )


2  2 2
1
= − ( A − A ' ) i.e., skew-symmetric matrix.
2
1 2  1 2   9 −4 
18. Ans. (d), ∵ A2 = 
 4 −3  4 −3  −8 17 
 = 

∴ f ( A ) = A2 + 4 A − 5 I
 9 −4   4 8  5 0 8 4 
 −8 17  16 −12   0 5  8 0 
=  + −  = 

19. Ans. (a), Since, A2 − B 2 = ( A − B )( A + B )


⇒ A2 − B 2 = A2 − B 2 + AB − BA ⇒ AB = BA
1 2   a 0   a 2b 
20. Ans. (c), AB = 
3 4   0 b  3a 4b 
 = 

Office.: 606 , 6 Floor, Hariom Tower, Circular Road, Ranchi-1, Ph.: 0651-2562523, 9835508812, 8507613968
th
4 ( ) BY R. K. MALIK’S NEWTON CLASSES
 a 0  1 2   a 2a 
and BA =   If AB = BA, then a = b
 0 b  3 4  3b 4b 
 =

Hence, AB = BA is possible for infinitely many values of B's.


1 1 1 1  2 2  1 1
21. Ans. (b), A2 =  = 2  = 2A
1 1 1 1  2 2  1 1
   = 

∴ A4 = A2 . A2 = 2 A.2 A = 4 A2 = 4 × 2 A = 23 A
A8 = A4 . A4 = 23 A.23 A = 26 A2 = 26.2 A = 27 A
Similarly, An = 2n −1 A ∴ A100 = 299 A
22. Ans. (c), A2 − 4 A + 10 I = A
 1 − 3  1 − 3  1 −3   1 0   1 − 3
−4 + 10 
2 k  2 k  2 k  0 1   2 k 
⇒      = 

 −5 −3 − 3k   4 −12  10 0  1 −3  1 9 − 3k  1 −3


 2 + 2k −6 + k 2  8 4k   0 10   2 k 
⇒  − + = ⇒  −6 + 2 k 4 + k 2 − 4k   2 k 
=

⇒ 9 − 3k = −3, − 6 + 2k = 2 ... ( i )
and 4 + k 2 − 4k = k ⇒ k 2 − 5k + 4 = 0 ⇒ k = 4, 1
But k = 1 is not satisfied the Eq. ( i ) .

23. Ans. (b), AAT = I = AT A

⇒ A AT = 1 = AT A ⇒ A2 = 1 ∵ AT = A 
 
⇒ A = ±1
 cos α sin α   cos α sin α 
24. Ans. (a), A2 (α ) = 
 − sin α cos α   − sin α cos α 
cos 2 α − sin 2 α 2 cos α sin α 
 −2sin α cos α cos 2 α − sin 2 α 
= 

 cos 2α sin 2α  ∵ cos 2 α − sin 2 α = cos 2α 


= A ( 2α )
 − sin 2α cos 2α   and 2sin α cos α = sin 2α 
=  

25. Ans. (b), Since, A, B and C are n × n matrices and, if A3 + 2 A2 + 3 A + 5 I = O, then A is invertible.
 1 1  1 1  1 1 1 1
 0 0

 2 2  2 2
 = 2 2 2 2

26. Ans. (c), A2 = 
  − −

 1 1  1 1 −1+1 1 1  0 0
 = 
 2 2 2 2

− 2 − 2 − 2 −
2  − +

  
∴ Matrix A is nilpotent of order 2.
3 2
27. Ans. (a), UV + XY = [ 2 − 3 4]  2  + [ 0 2 3]  2 
 
1   4 
= [ 6 − 6 + 4] + [ 0 + 4 + 12] = [ 4 ] + [ 16 ] = [ 20 ]
28. Ans. (c), A5 = A2 . A2 A = ( A + I )( A + I ) A
= ( A2 + 2 AI + I 2 ) A

Office.: 606 , 6 Floor, Hariom Tower, Circular Road, Ranchi-1, Ph.: 0651-2562523, 9835508812, 8507613968
th
( ) BY R. K. MALIK’S NEWTON CLASSES 5
= ( A + I + 2 A + I ) A = ( 3 A + 2I ) A
= 3 A2 + 2 IA = 3 ( A + I ) + 2 IA = 3 A + 3I + 2 A = 5 A + 3I
29. Ans. (d), m [ −3 4] + n [ 4 − 3] = [10 − 11]
⇒ [ −3m + 4n 4m − 3n ] = [10 − 11]
⇒ −3m + 4n = 10 ... ( i ) and 4m − 3n = −11 ... ( ii )
On solving Eqs. ( i ) and ( ii ) , we get n = 1, m = −2
Now, 3m + 7 n = 3 ( −2 ) + 7 (1) = 1
 1 2 3   −1 − 2
 −4 −5 − 6
30. Ans. (a), Given, P =  2 3 4   −2 0  
0 0 1
 3 4 5   0 − 4  
1 2 34 5 4 
=  2 3 4   8 10 12 
 3 4 5   0 0 −4 

5
∴ P22 = [ 2 3 4] 10  = 10 + 30 = 40
 0 

a g
31. Ans. (d), AB = [ x y z ]  h
h
b f 
 g f c 
= [ xa + yh + zg xh + yb + zf xg + yf + zc ]
Now, ABC = O
 x
⇒ [ ax + yh + zg xh + yb + zf xg + yf + zc ]  y  = O
 z 

⇒  ax 2 + hxy + gxz + hxy + y 2b + fzy + gxz + yfz + z 2 c  = O

⇒  ax 2 + by 2 + cz 2 + 2 gzx + 2hxy + 2 fyz  = O

6 8 5 6 4 9
32. Ans. (a), We have, A =  4 2 3  and A ' =  8 2 7 
 
 9 7 1   5 3 1 
A + A'
∴ Symmetric matrix, B =
2
  6 8 5   6 4 9   12 12 14   6 6 7 
1  1
=   4 2 3  +  8 2 7   =  12 4 10  =  6 2 5 
2  2  14 10 2   7 5 1 
  

  9 7 1   5 3 1      
0 α 0   α 2 0
33. Ans. (d), A =  2
α
1 1   1 1   α + 1 1 
= 

Office.: 606 , 6 Floor, Hariom Tower, Circular Road, Ranchi-1, Ph.: 0651-2562523, 9835508812, 8507613968
th
6 ( ) BY R. K. MALIK’S NEWTON CLASSES
2
0 1 0
∵ A2 = B ⇒ 
 α
1 1  5 1
= 
 α +
Which is not possible at the same time. ∴ No real values of α exists.
34. Ans. (b), ( aI + bA ) = ( aI + bA )( aI + bA )
2

2
= a 2 I 2 + aI ( bA ) + bA ( aI ) + ( bA )

Now, I 2 = I and IA = A ∴ ( aI + bA ) = a 2 I + 2abA + b 2 A2


2
( )
0 1 0 1  0 0 
Now, A2 =  = a 2 I + 2abA
2

0 00 0  0 0 
 = =O ∴ ( aI + bA )
35. Ans. (c), Since, the rank of given matrix is 1, then
2 5
= 0 ⇒ 2a − 8 + 20 = 0 ⇒ a = −6
−4 a − 4
 −3   4   10 
36. Ans. (a), Given x   + y   =  
 4  3   −5 
∴ −3 x + 4 y = 10 ... ( i )
and 4 x + 3 y = −5 ... ( ii )
On solving Eqs. ( i ) and ( ii ) , we get x = −2, y = 1
 4 1 0 0
37. Ans. (c), Let A =  3 0 1 0 
 6 0 2 0 
Now, we take a submatrix of order 3 × 3.
4 1 0
B =  3 0 1  ⇒ B = −1 ( 6 − 6 ) = 0
 6 0 2 
Now, we take a submatrix of order 2 × 2.
4 1
i.e., ⇒ C = 0−3 ≠ 0
3 0
C= 

∴ Rank of matrix A is 2.
 2 4 5 
38. Ans. (b), Given, A =  4
 8 10 
 −6 −12 −15
Applying R2 → R2 − 2 R1 and R3 → R3 + 3R1
2 4 5
⇒ A =  0 0 0 
 0 0 0 
Since, the equivalent matrix in echelon form has only one one-zero row,
∴ Rank ( A) = 1.

Office.: 606 , 6 Floor, Hariom Tower, Circular Road, Ranchi-1, Ph.: 0651-2562523, 9835508812, 8507613968
th
( ) BY R. K. MALIK’S NEWTON CLASSES 7

 1 −1 1 
39. Ans. (d), Let A =  1 1 −1
 −1 1 1 
∴ A = 1(1 + 1) + 1(1 − 1) + 1(1 + 1) = 4 ≠ 0 ∴ Rank of matrix A is 3.
 −1 2 5   0 0 a+6 
40. Ans. (b), A =  2 −4 a − 4  ∼  0 0 −a − 6

[ using R1 → R1 + R3 and R2 → R2 − 2 R3 ]
 1 −2 a + 1   1 −2 a + 1 

0 0 0 
∼  0 0 − a − 6  [ using R1 → R1 + R2 ]
 1 −2 a + 1 

 1 −2 a + 1 
∼  0 0 − a − 6 
 0 0 0 
 1 −2 7 
When a = 6 , A =  0 0 −12  ∴ ρ ( A) = 2
 0 0 0 
 1 −2 2 
When a = 1 , A =  0 0 −7  ∴ ρ ( A) = 2
 0 0 0 
 1 −2 3 
When a = 2, A =  0 0 −8  ∴ ρ ( A) = 2
 0 0 0 
x1 y1 1 x1 y1 1
41. Ans. (b), We have, x2 y2 1 = x2 − x1 y2 − y1 0 = 0 [ using R2 → R2 − R1 , R3 → R3 − R1 ]
x3 y3 1 x3 − x1 y3 − y1 0
∵ The given points ( x1 , y1 ) , ( x2 , y2 ) and ( x3 , y3 ) are collinear, therefore the rank of matrix is always
greater than 0 and less than 3.
1 0 1
42. Ans. (a), ∆ = 0 1 1 = −1 ≠ 0 , hence, matrix is non-singular.
1 0 0
43. Ans. (b), By inspection, A2 and A matrix is of order 3 × 3 , while B matrix is of order 3 × 2 .
Therefore, A2 + 2 B− 2 A not defined.
 1/ 3 2 3 6  1 0 
44. Ans. (b), We have, AB = 
 0 2 x − 3   0 −1   0 3 − 2 x 
   = 

1 0  1 0
[ AB = I (given)]
 0 3 − 2x   0 1 
⇒  = 

⇒ 3 − 2 x = 1 or x = 1 .
45. Ans. (d), It is obvious.

Office.: 606 , 6 Floor, Hariom Tower, Circular Road, Ranchi-1, Ph.: 0651-2562523, 9835508812, 8507613968
th
8 ( ) BY R. K. MALIK’S NEWTON CLASSES
1 λ 1 
46. Ans. (b), We have, A2 = A . A = 
λ
 −1 −λ   −1 −λ 
  

 λ 2 −1 0  0 0 
 0 −1 + λ 2 0 0 
⇒ =O= 

⇒ λ 2 −1 = 0 ⇒ λ = ±1 .
 0 1 −2 
47. Ans. (d), The matrix A =  −1 0 3  is singular.
 λ −3 0 
⇒ A = 0 ⇒ 0 − 1( −3λ ) + ( −2 )( 3) = 0 ⇒ 3λ − 6 = 0 ⇒ λ = 2
48. Ans. (c), A = k n B , by fundamental concept.
 3 −4   5 −8 
49. Ans. (d), X =  ⇒ X2 = 
 1 −1   2 −3 
 

 5 −8 
Clearly, for n = 2 , the matrices in (a), (b), (c) do not satisfy with  .
 2 −3 
50. Ans. (b), It is obvious.
51. Ans. (c), It is property of matrix multiplication.
52. Ans. (d), UV = [ 4] and XY = [16] ∴ UV + XY = [ 20] .
53. Ans. (a), We know that every identity matrix is a scalar matrix.
 x −2 
54. Ans. (d), Given A = 
3 7 

x −2
= 7x + 6
3 7
A =

  d −b  
1  7 2 ∵ If A =  a b  then A−1 =
  −c a  
∴ A−1 =
7 x + 6  −3 x 
 
 c d a b 
 
 
 c d 
7 1 
34 17  7 7

But given A−1 =  ⇒ 7 x + 6 = 34 ⇒ 7 x = 28 ⇒ x = 4

−3 2  7 x + 6 34
 ∴ =

34 17 



55. Ans. (b), Determinants of unit matrix of any order = 1 .


α 0 α 0   α2 0   1 0 
56. Ans. (d), A = 
2
(given)
 1 1  1 1   α + 1 1   5 1 
 = =

On comparing, we get,
α 2 = 1 and a + 1 = 5
⇒ a = ±1 and α = 4 , i.e., no real value of α .
57. Ans. (b), ∵ I −1 = I . [By theorem]

Office.: 606 , 6 Floor, Hariom Tower, Circular Road, Ranchi-1, Ph.: 0651-2562523, 9835508812, 8507613968
th
( ) BY R. K. MALIK’S NEWTON CLASSES 9
 1 0   10 0 
58. Ans. (a), By property A ( adj A ) = A I = 10 
 0 1   0 10 
= 

 3 2   4 −2   10 0 
Alternative method : A ( adj A ) =  .
 1 4   −1 3   0 10 
= 

59. Ans. (b), Since, the given matrix is symmetric, therefore a12 = a21
⇒ x + 2 = 2x − 3 ⇒ x=5
60. Ans. (c), (A− A )
T T
= AT − ( AT ) = AT − A ∵ ( AT )T = A
T

 
= − ( A − AT )
So, A − AT is a skew symmetric matrix.

61. Ans. (d), Given, A multiplicative group of 2 × 2 matrices of the form  .


a a
a a
 2 2
Let A =   since, A = 0, therefore, inverse of A does not exist.
 2 2
62. Ans. (c), Here, A2 = A
 2 −2 −16 − 4 x   2 −2 −4 
⇒  −1
 3 16 + 4 x  =  −1 3 4  On comparing, 16 + 4 x = 4 ⇒ x = −3
 4 + x −8 − 2 x −12 + x 2   1 −2 x 
1
63. Ans. (d), det ( AA−1 ) = det ( I ) ⇒ det ( A ) det ( A−1 ) = 1 ⇒ det ( A−1 ) =
det ( A)
64. Ans. (b), In skew-symmetric matrix all elements of diagonal are zero (always).
∴ Trace of A = 0 + 0 + 0 = 0
1 1 i 1  1 −i 
65. Ans. (b), ∵ A =
2 2 i a 
 −i a  ⇒ A =  
 
1 1 i
(Condition for unitary matrix)
2
∴ ( A )′ =  −i a  = A
θ
⇒ AAθ = I

1  1 i  1  1 i  1 0 1 2 i (1 + a )   1 0 
 −i a  =  0 1  ⇒ 2  −i 1 + a
2  2 ) 1 + a 2   0 1 
⇒  −i a  =
    (
∴ 1+ a = 0 ⇒ a = −1 (On comparing)
1 2 2 
66. Ans. (c), 3 A =  2 1 −2 
 x 2 y 

1 2 2 1 2 x 
∴ ( 3 A)( 3 A′ ) =  2 1 −2   2 1 2 
 x 2 y   2 −2 y 

 9 0 x + 2y + 4  1 0 0
= 0 9 2 x − 2 y + 2  = 9  0 1 0 

 x + 2 y + 4 2 x − 2 y + 2 x 2 + y 2 + 4   0 0 1 
On comparing, we get

Office.: 606 , 6 Floor, Hariom Tower, Circular Road, Ranchi-1, Ph.: 0651-2562523, 9835508812, 8507613968
th
10 ( ) BY R. K. MALIK’S NEWTON CLASSES
⇒ x + 2y + 4 = 0 …(1) and 2 x − 2 y + 2 = 0 …(2)
Solving (1) & (2), we get x = −2, y = −1 ∴ x + y = −3
67. Ans. (d), ∵ A = A, B = B
2 2
…(1) (given)
…(2) (given)
2
A + B = ( A + B ) = A2 + B 2 + AB + BA
⇒ A + B = A + B + AB + BA
⇒ AB + BA = O [using (1)]
1 2 3 1 2 3
68. Ans. (a), A =  2 4 6  =  0 0 0  [Operating R2 → R2 − 2 R1 and R3 → R3 − 3R1 ]
 3 6 9   0 0 0 
∴ ρ ( A) = 1 [∵ rank of matrix = no. of non-zeros row]
69. Ans. (c), A2 + A − I = O
⇒ A−1 ( A2 + A − I ) = A−1.O [pre-multiplication by A−1 ]

⇒ A + A−1 A − A−1 I = O ⇒ ( A + I ) − A−1 = O ⇒ A−1 = A + I

70. Ans. (c), By property, adj AT − ( adj A ) = O (null matrix)


T

71. Ans. (c), ρ ( A) ≤ min ( m, n ) i.e., r ≤ min ( m, n ) [by definition]


72. Ans. (b), AA′ = I , ∴ AA′ = I ⇒ A A =1 ∴ A = ±1
73. Ans. (b), ∵ A is orthogonal,
∴ AA′ = I ⇒ A−1 = A′ [pre-multiplication by A−1 ]

74. Ans. (a), If A = 


α β 

γ −α 
β   α 2 + βγ 0 
∴ A2 = 
α β  α
  = 2 
γ −α  γ −α   α α + βγ 
0 0 2
+ βγ = 0 )
0 0
= =O (∵ α
75. Ans. (c), ∵ AB ≠ BA
⇒ ( A + B ) ≠ A2 + 2 AB + B 2
2 2
∴ ( A + B) = A2 + AB + BA
1 0 0  1 0 0  1 0 0 
76. Ans. (d), A =  0
2
1 0   0 1 0  =  0 1 0  = I
 a b −1   a b −1   0 0 1 
77. Ans. (b), Given A is orthogonal
⇒ A A′ = 1 ⇒ A A = 1 ⇒ A =1
2
∴ AA′ = I ⇒ AA′ = I
∴ A = ±1

adj ( adj A ) = A (See formula)


( n −1)2

= ( ±1) = 1, when n is odd. (Note: Check this)


( n−1)2

Office.: 606 , 6 Floor, Hariom Tower, Circular Road, Ranchi-1, Ph.: 0651-2562523, 9835508812, 8507613968
th
( ) BY R. K. MALIK’S NEWTON CLASSES 11
 0 −c b − a ′ 
0 − a −b′ 
78. Ans. (b), Let A =
 c
0 − c′ 

 −b a
0 
 
 a′ b′ c′
Applying R4 → aR4
 0 −c b − a′ 
 c 0 −a −b′ 
0
A=
 −b a − c′ 
0 
 
 aa′ ab′ ac′
Applying R4 → R4 − b′R3
 0 −c b − a′ 
 0 − a −b′ 
0 −c ′ 
 c
A=
 −b
 aa′ + bb′ 0 ac′ b′c′ 
a
 

Applying R4 → R4 + c′R2 , then


 0 −c b − a ′ 
0 − a −b′ 
[∵ aa′ + bb′ + cc′ = 0]
 c
0 − c′ 
A= 
 −b a
 0 0 0 0 
 

Applying R3 → cR3
 0 −c b − a ′ 
 c 0 − a −b′ 
 −bc ac 0 −cc′ 
A= 

 0 0 0 0 
 

Applying R3 → R3 + aR1 + bR2 , then we get


 0 −c b − a′ 
 c 0 − a − b′ 
0 0
[∵ aa′ + bb′ + cc′ = 0]
0 0 
A= 

0 0 0 0 
 

Number of non-zero rows = 2 ∴ ρ ( A) = 2

79. Ans. (d), ∵ ( A + B ) = A2 + AB + BA + B 2


2

3 2
∴ ( A + B) = ( A + B )( A + B ) = ( A + B ) ( A2 + AB + BA + B 2 )

= A3 + A ( AB ) + A ( BA) + AB 2 + BA2 + B ( AB ) + B ( BA) + B3


⇒ Number of distinct terms = 8
1
80. Ans. (b), We know that adj A−1 = A−1 …(1)
2
2
=
A
1
Also, ( adj A ) −1 −1
= A = 22 = 4 …(2) [using (1)]
2

adj A
= −1

Office.: 606 , 6 Floor, Hariom Tower, Circular Road, Ranchi-1, Ph.: 0651-2562523, 9835508812, 8507613968
th
12 ( ) BY R. K. MALIK’S NEWTON CLASSES
 a 0
81. Ans. (b), Since A = 
0 b

 a 0  a 0   a 2 0
⇒ A2 = 
 0 b  0 b   0 b2 
 = 

 a 2 0   a 0   a3 0
⇒ A3 = A2 . A = 
0 2 
0 0
=
b3 

 b   b 
…….. ……..
 an 0
0
∴ An =  
bn 
−1 1  bn 0   a−n 0 
a nb n  0 an   0
⇒ ( An ) =  = 
b−n 
1
 lim a − n 0   lim 0
1 1
 
⇒ lim A n −1
[∵ as n → ∞, → 0, n → 0 ]

 0 lim b 1 
n →∞ a n
( )=  n→∞  = 
  0 lim n 
−n
n →∞  a n
b
 n →∞
 n →∞ b 

0 0
∴ lim ( An ) =   as a > 1 and b > 1 .
−1

n →∞
0 0
82. Ans. (d), det ( A − I ) = det ( A − A2 ) ∵ A2 = I 

= det { A ( I − A)} = det A det ( I − A )


= − det A det ( A − I )
⇒ det ( A − I ){1 + det A} = 0 …(i)
Now A2 = I
⇒ det ( A2 ) = det I ⇒ ( det A ) = 1
2

∴ det A = 1 (∵ A > 0 )

Hence (i) ⇒ A − I = 0

83. Ans. (c), adj ( Q −1 BP −1 ) = adj ( P −1 ) .adj ( B ) .adj ( Q −1 )

. A. ∵ adj ( P ) =
−1
P Q  P 
= 
P Q  P 

= PAQ ∵ P = Q = 1


1+ x
84. Ans. (c), ∵ f ( x ) =
1− x
⇒ (1 − x ) f ( x ) = (1 + x ) ⇒ ( I − A) f ( A) = ( I + A)
−1
f ( A ) = ( I − A) ( I + A)
  1 0   1 2     1 0   1 2    0 −2   2 2 
−1 −1

  0 1   2 1     0 1   2 1    −2 0   2 2 
= −   +  =    

Office.: 606 , 6 Floor, Hariom Tower, Circular Road, Ranchi-1, Ph.: 0651-2562523, 9835508812, 8507613968
th
( ) BY R. K. MALIK’S NEWTON CLASSES 13

1 0 2 2 2  1  4 4   −1 −1 
4 2 0  2 2  4  4 4   −1 −1 
=−     =−  =

85. Ans. (b), Since, AB = BA


1 2  a b   a b  1 2
3 4  c d   c d 
⇒   =  3 4
 
 a + 2c b + 2 d   a + 3b 2a + 4b 
 =  c + 3d 2c + 4d  , then a + 2c = a + 3b
 3a + 4c 3b + 4d
⇒ 
  
⇒ 2c = 3b ⇒ b≠0 (∵ c ≠ 0 )
b + 2d = 2a + 4b ⇒ 2a − 2d = −3b
3b
3b 2 = −1
or, ( a − d ) = − (∵ 2c = 3b )

3b − c 3b − 3b
a−d
2
∴ =

2
86. Ans. (c), Since,
1  1  1  1 
 A − I   A′ − I ′  = I and  A + I  A′ + I ′  = I
2  2  2  2 
 
 
1 1 1 1
⇒ AA′ − I ( A + A′ ) + I = I and AA′ + I ( A + A′ ) + I = I
2 4 2 4
both conditions are true only when A + A′ = 0
3 3
⇒ A′ = − A and AA′ + = I ⇒ AA′ = I ⇒ − A2 = I
4 4 4
I

3 3 3  3
(Assume order of matrix is n )
2 2 2
n

4 4 4  4
∴ A =− I ⇒ A = − I =  ⇒ A = − 

⇒ n is even.
87. Ans. (d), Since, P′P = I …(1)
Q = PAP′
⇒ P′Q = P ' PAP′ (On pre-multiplication by P ' )
∴ P ' Q = AP ' …(2) [using (1)]
and X = P′Q 2010 P = P′Q ( Q 2009 P )

= AP′ ( Q 2009 P ) [using (2)]

= A ( P′Q ) ( Q 2008 P )
= A2 ( P′Q ) ( Q 2007 P ) [using (2)]

= A3 P′ ( Q 2007 P ) [using (2)]


………………………..
= A2009 P′ ( Q ) P = A2009 ( AP′ ) P = A2010 ( P′P ) = A2010 ( I ) [using (1)]
= A2010 = I (∵ A is orthogonal)
Hence, X −1 = I .

Office.: 606 , 6 Floor, Hariom Tower, Circular Road, Ranchi-1, Ph.: 0651-2562523, 9835508812, 8507613968
th
14 ( ) BY R. K. MALIK’S NEWTON CLASSES
 1   2 1 −1 
88. Ans. (c),  −1  [ 2 1 −1 ] =  −2 −1 1  .
 
 2   4 2 −2 
89. Ans. (c), For subtraction of two matrix, they should be of the same order, i.e., p = r , q = s .
90. Ans. (c), ∵ AB = O ⇒ AB = 0
⇒ A B =0 ⇒ A = 0 or B = 0 .
When AB = O , neither A nor B may be O .
1 0 0 0 0 0
For example if A =  and B =  , then AB =  .
0 0 1 0 0 0
 

91. Ans. (b), We know that Adj A = A = d n −1 . [∵ A = d ]


n −1

92. Ans. (b), The given matrix is a skew-symmetric matrix because A′ = − A .


93. Ans. (c), We know that A . adj A = A A for order n
n −1 n
= A
Hence, DD′ = D n .
0 1 −1
94. Ans. (a), We have, D = −1 0 2 =0
1 −2 0
1 1 −1
D1 = −2 0 2 = 14 ⇒ D1 ≠ 0
3 −2 0
∵ D = 0 and D1 ≠ 0 , hence the given is inconsistent, so it has no solution.
95. Ans. (d), We have, M 2 − λ M − I 2 = 0
1 21 2  λ 2λ   1 0 
=0 (On putting value of M )
 2 3   2 3   2λ 3λ   0 1 
⇒  − −

 5 8   λ 2λ   1 0   4 − λ 8 − 2λ   0 0 
=0
 8 13   2λ 3λ   0 1   8 − 2λ 12 − 3λ   0 0 
⇒  −  −  ⇒ = 

∴ λ = 4. (On comparing)
96. Ans. (c), Definition: If all the elements below the diagonal in the matrix are zero, then it is an upper
triangular matrix.
 0 3a 
97. Ans. (c), Given, kA = 
 2b 24 

 0 2   0 3a 
⇒ 2k = 3a, 3k = 2b, − 4k = 24
 3 −4   2b 24 
⇒ k = 

2k 3k
, b = , k = −6 ⇒ k = −6, a = −4, b = −9
3 2
⇒a=

98. Ans. (a), adj ( A) can be obtained by changing the principle diagonal elements and changing the sign
of sub-diagonal elements. (Only for second order matrices)
 3 1
Here, adj ( A ) =  .
 −2 1 

Office.: 606 , 6 Floor, Hariom Tower, Circular Road, Ranchi-1, Ph.: 0651-2562523, 9835508812, 8507613968
th
( ) BY R. K. MALIK’S NEWTON CLASSES 15
99. Ans. (b), A = ( ad − bc )
1
∴ A−1 = .
 d −b 
( ad − bc )  −c a 
100. Ans. (d), A = 1 ≠ 0 , therefore, A is invertiable.
Thus, (d) is not correct.
101. Ans. (a), ( B −1 A−1 ) = ( A−1 )
−1 −1 −1 −1
(B ) = AB

 2 2   0 −1   2 −2 
.
 −3 2   1 0   2 3 
=  =

102. Ans. (c), ∵ AAT = I


∴ AAT = I

A A =1 ∴ A = ±1 .

103. Ans. (b), ∵


2
( A + B) = A2 + B 2
⇒ A2 + B 2 + AB + BA = A2 + B 2
 1 −1   a 1   a 1   1 −1 
⇒ AB + BA = 0
 2 −1   b −1   b −1   2 −1 
⇒   +  =O

 a − b 2   a + 2 −a − 1   2a − b + 2 − a + 1   0 0 
=0 ⇒ 
 2a − b 3   b − 2 −b + 1   2a − 2 −b + 4   0 0 
⇒   +  =

On comparing, we get,
2a − b + 2 = 0, − a + 1 = 0, 2a − 2 = 0, − b + 4 = 0 ∴ a = 1, b = 4 .
 0 −1 
104. Ans. (d), Given, matrix A =  .
1 0 
 0 −1   0 −1   −1 0 
We know that A2 = A . A =  .
 1 0   1 0   0 −1 
 =

 −1 0   ( −1) 0  1 0
8 8

Therefore, A = ( A
16 2 8
.
 0 −1   0 ( −1)   0 1 
) =  = 8
 =

105. Ans. (d), On expansion, A = k 2 + 1 , which can be never zero.


Hence, matrix A is invertible for all real k .
3 2 4
106. Ans. (d), We know, k = A ⇒ k = 0 18 0 = 11
0 0 18
∴ k = 11 .
1 1 1
107. Ans. (a), ∵ A−1 =
( adj ( A) ) = λ
A ( 0 − 6) 6
A
( adj ( A) ) ⇒ λ= = =−

 10 0 
108. Ans. (b), We have, A ( adj A ) = 
 0 10 

1 0
⇒ A ( adj A ) = 10   = 10 I …(1)
0 1

Office.: 606 , 6 Floor, Hariom Tower, Circular Road, Ranchi-1, Ph.: 0651-2562523, 9835508812, 8507613968
th
16 ( ) BY R. K. MALIK’S NEWTON CLASSES
1
and, A−1 = [∵ AA−1 = I ]
A ( adj A )
( adj A) ⇒ =I
A A
10I
⇒ =I ⇒ A = 10 [using (1)]
A
2 3 1 4 
109. Ans. (a), Applying R3 → R3 + 2 R2 , then A =  0 1 2 −1 
 0 0 0 0 
ρ ( A) = No. of non zero rows in A = 2
 1 2 2 
1
110. Ans. (a), Since for given A =  2 1 −2 
3
 −2 2 1 
AAT = AT A = I ( 3×3) , thus, A is orthogonal.
111. Ans. (c), It is obvious.
 4 2 2
112. Ans. (a), Given,  −5 0 α  = 10 A−1
 
 1 −2 3 
 
 4 2 2
⇒  −5 0 α  A = 10 A−1 A = 10 I [∵ A−1 A = AA−1 = I ]
 
 1 −2 3 
 
 4 22  1 −1 1   10 0 0 
⇒  −5 0 α  2 1 −3  =  0 10 0 
    
 1 −2 3 
 1 1 1   0 0 10 
   
⇒ −5 + α =0 ⇒ α =5
4 1 0 0
113. Ans. (c), Applying, R3 → R3 − 2 R2 , then  3 0 1 0 
 0 0 0 0 
∴ Required rank is 2.

1 −1 1
114. Ans. (d), We have, A = 0 2 −3 = 1 ( 0 + 3 ) + 1 ( 0 + 6 ) + 1 ( 0 − 4 ) = 5
2 1 0
 3 1 1 
Since, B = ( adj A ) =  −6 −2 −3  ,
 −4 −3 2 

 5 −5 5 
( adj B ) =  0 10 −15  = 5 A
 10 5 0 
Now comparing C = 5 A , We get

Office.: 606 , 6 Floor, Hariom Tower, Circular Road, Ranchi-1, Ph.: 0651-2562523, 9835508812, 8507613968
th
( ) BY R. K. MALIK’S NEWTON CLASSES 17

C = ( adj B ) ⇒ C = ( adj B ) ;

=1
( adj B )

C
115. Ans. (c), By fundamental property, adj ( λ X ) = λ n −1 ( adj X ) .
∴ adj ( λ X ) = λ 3−1 ( adj X ) [∵ order of matrix, n = 3 ]
⇒ adj ( λ X ) = λ 2 ( adj X ) .

116. Ans. (b), Let   be the identity element then,


a a
a a
x x  a a   x x  2ax 2ax   x x
 2ax 2ax   x
  =  ⇒  = 
x x  a a   x x x
1 11 1
⇒ 2ax = x (∵ x ≠ 0 ) .
∴ Identity element =  .
2 21 1
⇒ a=

117. Ans. (d), The given equation can be written into matrix from.
 1 1 −1   x   0 
 3 −1 −1   y  =  0  ⇒ AX = 0 Which is homogeneous equations.

 1 −3 1   z   0 
    

1 1 −1
∆ = 3 −1 −1 = 1( −1 − 3) − 1( 3 + 1) − 1( −9 + 1) = − 4 − 4 + 8 = 0
1 −3 1
∴ There are infinite number of solutions.
118. Ans. (b), By definition it is obvious.
 0 1 2
Example : Let A =  −1 0 3 
 −2 −3 0 

 0 −1 −2   0 1 2
⇒ A′ =  1 0 −3  = −  −1 0 3  = − A .
 
 2 3 0   −2 −3 0 
which shows that A is skew-symmetric.
b 2   ab b2 
119. Ans. (a), A = A. A =  2
2  ab
− ab   − a 2
 
 −a − ab 
 a 2 b 2 − a 2b 2 ab3 − ab3 
3 3
− a 2b 2 + a 2b 2
=  =O
 −a b + a b 
⇒ A3 = A. A2 = O and An = O, for all n ≥ 2 .
120. Ans. (d), It is obvious.
121. Ans. (c), Since a square matrix A whose elements aij = 0 for i < j .
Then A is the lower triangular matrix.
122. Ans. (b),  aij  square matrix is a upper triangular matrix for aij = 0, i > j .
123. Ans. (d), Let AB = BA
n× n

Office.: 606 , 6 Floor, Hariom Tower, Circular Road, Ranchi-1, Ph.: 0651-2562523, 9835508812, 8507613968
th
18 ( ) BY R. K. MALIK’S NEWTON CLASSES
1 2 a 0 a 01 2   a 2b   a 2a 
3 4 0 b   0 b 3 4   3a 4b   3b 4b 
⇒   =   ⇒ = 

a 0
This is only possible, when a = b . So, B should be of the form  .
0 a
As, a ∈ N so there are infinitely many B’s.
124. Ans. (b), It is based on fundamental concept.
 2 0 0   2 0 0   −1 0 0 
125. Ans. (b), A B = ( A. A ) B =  0 −1 0   0 −1 0   0 3 0 
2

 0 0 3   0 0 3   0 0 2 

 4 0 0   −1 0 0   −4 0 0 
=  0 1 0   0 3 0  =  0 3 0 
 0 0 9   0 0 2   0 0 18 

126. Ans. (c), AC = [ a b ]  =  a 2 − ab 


 a 

 −a 

=  a 2 − ab 
 a 
BC = [ −b − a ]  
 −a 
∴ AC = BC
127. Ans. (a), We have ( A + B )( A − B ) = A2 − AB + BA − B 2
∴ Option (a) is not true.
128. Ans. (b), Order will be (1× 3)( 3 × 3)( 3 × 1) = (1×1) .
129. Ans. (b), A + B is defined ⇒ A and B are of same order.
Also AB is defined ⇒ Number of columns in A = Number of rows in B
Obviously, both simultaneously mean that the matrices A and B are square matrices of same order.
130. Ans. (b), = a 2 I 2 + b 2 A2 + 2ab AI
2
( aI + bA)
= a 2 I 2 + b 2 A2 + 2abA
0 0
But A2 =  ∴ ( aI + bA ) = a 2 I + 2abA
2

0 0

131. Ans. (a), We know that every identity matrix is a scalar matrix.
 1 1
132. Ans. (c), Let matrix A =  is commutative with B = 
a b
 0 1
 
c d 
 a b  1 1  1 1 a b 
 c d  0 1  0 1 c d 
⇒ AB = BA ⇒  =  

 a a+b   a+c b+d 


⇒  =
 c c+d   c d 
∴ a = a + c ⇒ c = 0 and a + b = b + d ⇒ a=d.
 1   2 1 −1 
133. Ans. (c),  −1  [ 2 1 −1 ] =  −2 −1 1 
 2   4 2 −2 

Office.: 606 , 6 Floor, Hariom Tower, Circular Road, Ranchi-1, Ph.: 0651-2562523, 9835508812, 8507613968
th
( ) BY R. K. MALIK’S NEWTON CLASSES 19
 1 −2 
134. Ans. (b), Given that, A =  and f ( t ) = t 2 − 3t + 7
4 5 

 1 −2   1 −2   −7 −12 
Now, A2 = 
 4 5   4 5   24 17 
 = 

∴ f ( A ) = A2 − 3 A + 7
 −7 −12   1 −2   1 0   −3 −6 
−3 +7
 24 17  4 5   0 1   12 9 
=   = 

 3 6   −3 −6   3 6  0 0
 −12 −9   12 9   −12 −9   0 0 
∴ f ( A) +   =  + = 

1 1 01 1 0 2 3 1
134. Ans. (b), A = AA =  1 2 1   1 2 1  =  5 6 2 
2

 2 1 0   2 1 0   3 4 1 

 2 3 1 1 1 0   7 9 3 
⇒ A = A A =  5 6 2   1 2 1  =  15 19 6 
3 2

 3 4 1   2 1 0   9 12 4 
1 0 0
Here, A3 − 3 A2 =  0 1 0  = I ⇒ A3 − 3 A2 − I = 0
 0 0 1 
 1 −1  a 1 
135. Ans. (b), Given A =  , B=
 2 −1   b −1 
 

 1+ a 0 
 2 + b −2 
⇒ A+ B =  

 1 −1   1 −1   −1 0   a 1   a 1   a2 + b a −1 
A2 =  ; B2 = 
 2 −1   2 −1   0 −1   b −1   b −1   ab − b b + 1 
 =  = 

 a2 + b −1 a −1 
A2 + B 2 =  
 ab − b b 

 1+ a 0   1+ a 0  1+ a) 0
2

Also, ( A + B )
2 2  (
 2 + b −2   2 + b −2   ( 2 + b )(1 + a ) − 2 ( 2 + b ) 4 
=   ⇒ ( A + B) = 

2 2 2
 (1 + a )2 0  a2 + b −1 a −1 
( 2 + b )( a − 1) 4   ab − b
∴ ( A + B) = A +B ⇒ = 
b 
By equation, a − 1 = 0 ⇒ a = 1 and b = 4
Alternative : ( A + B )( A + B ) = A2 + AB + BA + B 2 = A2 + B 2
∴ AB + BA = 0 ⇒ AB = − BA
Check for AB and BA we get option (b).
136. Ans. (a), It is obvious.
137. Ans. (b), Since A, B are symmetric ⇒ A = A′ and B = B′

∴ ( AB − BA)′ = ( AB )′ − ( BA )′ = B′A′ − A′B′ = − ( A′B′ − B′A′ ) = − ( AB − BA )

Office.: 606 , 6 Floor, Hariom Tower, Circular Road, Ranchi-1, Ph.: 0651-2562523, 9835508812, 8507613968
th
20 ( ) BY R. K. MALIK’S NEWTON CLASSES
⇒ ( AB − BA) is skew-symmetric.
138. Ans. (b), Given, AB = 3I
⇒ A−1 ( AB ) = A−1 ( 3I ) [Pre-multiplication by A−1 ]

⇒ A−1 AB = 3 A−1 I ⇒ IB = 3 A−1 ∵ A−1 A = I 


1
⇒ B = 3 A −1 ⇒ A−1 = B
3
139. Ans. (c), ( A + B )( A − B ) = A2 − B 2 , this is possible,
if , AB = BA
−1 −1 2 2
∴ ( ABA ) = ( BAA ) = ( BI ) = B2

140. Ans. (d), Since ( A + A′ )′ = A′ + A = A + A′, so it is symmetric.

( AA′ )′ = ( A′ )′ A′ = AA′, so it is symmetric

( A′A )′ = A′ ( A′)′ = A′A, so it is symmetric.

But ( A − A′ )′ = A′ − A ≠ A − A′. Hence it is not symmetric.


141. Ans. (a), 3 A3 + 2 A2 + 5 A + I = 0 ⇒ I = −3 A3 − 2 A2 − 5 A
⇒ IA−1 = −3 A2 − 2 A − 5 I [post-multiply by A−1 in both sides]
⇒ A−1 = − ( 3 A2 + 2 A + 5 I )

142. Ans. (d), aij = i 2 − j 2 is a square matrix. For a skew symmetric matrix aij = − a ji
⇒ aij = i 2 − j 2 and a ji = j 2 − i 2 ⇒ aij + a ji = 0 ⇒ aij = − a ji
Hence, aij is a skew symmetric matrix.
1 0 0
143. Ans. (c), adj ( I 3 ) =  0 1 0  and I 3 = 1
 0 0 1 
adj ( I 3 )
∴ I 3−1 = = I3
I3

144. Ans. (a), [Reversal law of inverse]


−1 −1 −1 −1 −1
(B A − 1 ) = ( A− 1 ) (B ) = AB

 2 2   0 −1   2 −2 
.
 −3 2   1 0   2 3 
=  =

145. Ans. (c), A = 1( 27 × 4 − 9 × 8) − 2 ( 27 − 9 ) + 3 ( 8 − 4 )


= 36 − 36 + 3 × 4 = 12
we know that, adj A = A
n −1

= A = (12 ) = 144 .
3−1 2 2
∴ adj A = A
146. Ans. (d), We have aij = i 2 − j 2
∴ a ji = j 2 − i 2 ⇒ aij = − a ji
Thus, A is skew-symmetric matrix of even order.

Office.: 606 , 6 Floor, Hariom Tower, Circular Road, Ranchi-1, Ph.: 0651-2562523, 9835508812, 8507613968
th
( ) BY R. K. MALIK’S NEWTON CLASSES 21
We know that the determinant of every skew-symmetric matrix of even order is a prefect square that of
odd order is zero. Hence, option (d) is correct.
147. Ans. (b), We know that ( AT ) = ( A−1 )
−1 T

So, option (a) is true.


In option (b), A−1 is matrix and A is a number.
−1

So, it is not true.


Now, A2 ( A−1 ) = ( AA ) ( A−1 A−1 )
2

2
⇒ A2 ( A−1 ) = A ( AA−1 ) A−1 = ( AI ) A−1 ∵ AA−1 = I 

= AA−1 = I
2 −1
= ( A−1 ) . So, option (c) is true.
2
∴ (A )
1 0 0 0 0 0
148. Ans. (a), If A =  , B= , then AB =  .
0 0 0 1 0 0
 

However, A ≠ O, B ≠ O . So, option (a) is correct.


149. Ans. (c), Since A is a diagonal matrix. Therefore
A = a × a × a = a3

Also, adj A = A
3−1
∵ adj A = A n −1

 
⇒ adj A = A = a 6
2

150. Ans. (c), We have, B −1 AB = B −1 A B

1 1 
⇒ B −1 AB = −1

A B ∵ B = 
B  B 

⇒ B −1 AB = A

151. Ans. (a), It is given that A ≠ 0 . So, A−1 exists.


Now, AB = O
⇒ A−1 ( AB ) = A−1O

⇒ ( A−1 A ) B = O ⇒ IB = O ⇒ B=O
1 + 2i 
152. Ans. (d), We have, A = 
 −i
 −1 + 2i 0 
 −i −1 + 2i   i 1 − 2i 
 1 + 2i 0   −1 − 2i 0 
( A)
T
=  = − =−A

Hence, A is skew-hermitian.
153. Ans. (d), We have, A2 = B
α 0 α 0 1 0  α2 0  1 0 
1 1   1 1   5 1   α +1 1   5 1 
⇒  = ⇒ = 

⇒ α 2 = 1 and α + 1 = 5
Clearly, these two equations are inconsistent.
Hence, there is no value of α satisfying A2 = B .

Office.: 606 , 6 Floor, Hariom Tower, Circular Road, Ranchi-1, Ph.: 0651-2562523, 9835508812, 8507613968
th
22 ( ) BY R. K. MALIK’S NEWTON CLASSES
154. Ans. (c), Clearly, the element in the first row and third column of the inverse of the given matrix is co-
factor of the element in third row and first column of the given matrix.
2 −3
Hence, required element = =7
1 2
 a b3   1 8 
155. Ans. (d), We have,  ⇒ a = 1, b3 = 8 ⇒ a = 1, b = 2
2 0  2 0
= 

a b 1 2
2 0 2 0
∴  = 

1  0 −2   0 1/ 2 
−1

Hence, 
a b
2 0 4  −2 1   1/ 2 −1/ 4 
 =−   =

156. Ans. (a), We have A2 − A + I = O

⇒ A − A2 = I ⇒ A−1 = I − A
I
⇒ A ( I − A) = I ⇒ I −A=

157. Ans. (c), We have, B = 64


A

⇒ adj A = 64 ⇒ A = 64 ∵ adj A = A , where n is order of matrix A


2 n −1
 
⇒ A = ±8
158. Ans. (b), Since A is an orthogonal matrix.
∴ AAT = I
1 2 2  1 2 a 1 0 0
1 1
⇒  2 1 −2  .  2 1 2  =  0 1 0 
3 3
 a 2 b   2 −2 b   0 0 1 

1 2 2 1 2 a  1 0 0 
1
⇒  2 1 −2   2 1 2  =  0 1 0 
9
 a 2 b   2 −2 b   0 0 1 

 9 0 a + 4 + 2b   9 0 0 

⇒  0 9 2a + 2 − 2b  =  0 9 0 
 a + 4 + 2b 2a + 2 − 2b a 2 + 4 + b 2   0 0 9 
⇒ a + 4 + 2b = 0, 2a + 2 − 2b = 0 and a 2 + 4 + b 2 = 9
⇒ a + 2b + 4 = 0, a − b + 1 = 0 and a 2 + b 2 = 5
⇒ a = −2, b = −1
1 0
159. Ans. (a), A = ω 
0 1
 = ωI2

= ω100 I 2 = (ω 3 ) .ω I 2 = ω I 2 ∵ ω 3 = 1


100 33
∴ A100 = ω100 ( I 2 )
= ω I2 = A
0 1 −1
160. Ans. (a), We have A = 2 1 3 = 0 + 7 −1 = 6
3 2 1

Office.: 606 , 6 Floor, Hariom Tower, Circular Road, Ranchi-1, Ph.: 0651-2562523, 9835508812, 8507613968
th
( ) BY R. K. MALIK’S NEWTON CLASSES 23

∴ ( A ( adj A) A ) A = ( A ( adj A) ) ( A A)
−1 −1

⇒ ( A ( adj A) A ) A = ( A I ) I
−1
[∵ A ( adj A ) = A I n where n is order of matrix A ]

6 0 0 3 0 0
⇒ ( A ( adj A ) A ) A = A I =  0 6 0  = 2  0 3 0 
−1  
 0 0 6   0 0 3 
Hence, option (a) is correct.
161. Ans. (a), If A is a non-singular matrix of order n, then adj ( adj A ) = A
n−2
A
Here, n = 3 ∴ adj ( adj A ) = A A
162. Ans. (b), We have
A an orthogonal matrix ⇒ A = ±1 [∵ determinant at orthogonal matrix = ± 1 ]
B a skew-symmetric matrix of odd order
⇒ B =0 [∵ determinant of skew symmetric matrix at odd order will be zero]
∴ AB = A B ⇒ AB = ( ±1) × 0 = 0
163. Ans. (d), We have, A = 1 ≠ 0
So, A−1 exists.
Clearly, A ≠ ( −1) I and A is not a zero or null matrix.
So, options (a), (b) and (c) are not correct.
 0 0 −1   0 0 −1   1 0 0
Now, A =  0 −1 0   0 −1 0  =  0 1
2
0  = I
 −1 0 0   −1 0 0   0 0 1 
Hence, option (d) is correct.
164. Ans. (d), It is given that B is the inverse of matrix A.
∴ AB = I
 1 −1 1   4 2 2   10 0 0 
⇒ A (10 B ) = 10 I ⇒  2 1 −3   −5 0 α  =  0 10 0 
 1 1 1   1 −2 3   0 0 10 
 10 0 5 − α   10 0 0 
⇒  0 10 α − 5  =  0 10 0  ⇒ α =5
 0 0 α + 5   0 0 10 
Alternative : We have, AB = I
⇒ A (10 B ) = 10 I ⇒ ( A (10 B ) )13 = 0

2 
⇒ [ 1 −1 1 ]  α =0 ⇒ 5 −α = 0 ⇒α =5
 3


165. Ans. (a), We have
A2 = A and B = I − A
2
∴ AB + BA + I − ( I − A ) = A ( I − A ) + ( I − A) A + I − ( I − A )( I − A)

Office.: 606 , 6 Floor, Hariom Tower, Circular Road, Ranchi-1, Ph.: 0651-2562523, 9835508812, 8507613968
th
24 ( ) BY R. K. MALIK’S NEWTON CLASSES
= A − A2 + A − A 2 + I − ( I − 2 A + A 2 ) [∵ IA = A]
= A − A + A − A + I − ( I − 2 A + A) ∵ A2 = A
=A
166. Ans. (a), a11 A11 + a12 A12 + a13 A13
2 1 1 1 1 2
=3 −2 +4
2 6 3 6 3 2
= 3 (12 − 2 ) − 2 ( 6 − 3) + 4 ( 2 − 6 ) = 30 − 6 − 16 = 8
 3 5   1 17   3 1  3 1
167. Ans. (b), AB =  = 100
 2 0   0 −10   2 34  2 34
 =
   ⇒ AB =

OR AB = A B = ( −10 )( −10 ) = 100

168. Ans. (b), ∵ det ( A1 ) = = a2 − b2


a b
b a
a2 b2
det ( A2 ) = = a 4 − b4
b2 a 2

∑ det ( A ) = det ( A ) + det ( A ) + .....



∴ 1 2
i =1
i

= a 2 − b 2 + a 4 − b 4 + ..... ( a 2 + a 4 + .....∞ ) − ( b 2 + b 4 + .....∞ )

a2 b2 a2 − b2
1 − a 2 1 − b 2 (1 − a 2 )(1 − b 2 )
= − =

169. Ans. (d), B = 5 A2


⇒ det ( B ) = det ( 5 A2 ) = 53  det ( A )  ∵ O ( A ) = 3
2

= 125 ( 6 ) = 4500 [give det A = 6 ]


2

2 1 0
170. Ans. (d), A =  0 2 1 
 1 0 2 
C11 = 4, C12 = 1, C13 = −2 ; C21 = −2, C22 = 4, C23 = 1 ; C31 = 1, C32 = −2, C33 = 4

 4 1 −2   4 −2 1 
T

∴ adj ( A ) =  −2 4 1  =  1 4 −2 
 1 −2 4   −2 1 4 
∴ adj A = 4 (16 + 2 ) + 2 ( 4 − 4 ) + (1 + 8) = 72 + 0 + 9 = 81
Alternative :
2 1 0
A = 0 2 1 = 2 ( 4 − 0 ) − 1( 0 − 1) = 9
1 0 2

∴ adj A = A = ( 9 ) = 81 ∵ adj A = A where n is the order of matrix A


3−1 2

Office.: 606 , 6 Floor, Hariom Tower, Circular Road, Ranchi-1, Ph.: 0651-2562523, 9835508812, 8507613968
th
( ) BY R. K. MALIK’S NEWTON CLASSES 25
171. Ans. (b), Since, AB = 3I

⇒ A−1 AB = 3IA−1 ⇒ B = 3 A−1 ⇒ A−1 =


3
B

  d −b  
1  1 −2  ∵ If A =  a b  then A−1 =  −c a 
172. Ans. (a), A−1 =
   

1 + 10  5 1 

 c d a b 
 
 
 c d 
1  1 −2 
11  5 1 
=

Also, A−1 = xA + yI
1  1 −2   x 2 x   y 0   x + y 2 x 
11  5 1   −5 x x   0 y   −5 x x + y 
⇒ = + =

1 −2 −1 2
⇒ x + y = , 2x = ⇒ x= , y=
11 11 11 11
173. Ans. (d), C11 = 3, C12 = −6, C13 = −4, C21 = 1, C22 = −2, C23 = −3, C31 = 1, C32 = 3, C33 = 2
 3 −6 −4   3 1 1 
Hence ( adj A ) =  1 −2 −3  =  −6 −2 3 
 1 3 2   −4 −3 2 

 3 1 1  5 −5 5 
B = adj ( A) =  −6 −2 3  . Similarly, adj ( B ) =  0 10 −15 
 −4 −3 2   10 5 0 
5 −5 5 1 −1 1
Now, adj B = 0 10 −15 = 625 and C = 125 A = 125 0 2 −3 = 125 × 5 = 625
10 5 0 2 1 0
adj ( B ) 625
=1
625
∴ =
C
Alternative :
A = 1 ( 0 + 3 ) + 1( 0 + 6 ) + 1( 0 − 4 ) = 5

Now, adj B = adj ( adj A) = A A ∵ adj ( adj A) = A . A where n is order of matrix A


n−2
 
= 5A .
adj B 5A
=1
5A
∴ =
C
 1 0 2   5 a −2 
174. Ans. (c), Given adj  −1 1 −2  =  1 1 0  …(i)
 0 2 1   −2 −2 b 

 1 0 2 
Cofactors of  −1 1 −2 
 0 2 1 

Office.: 606 , 6 Floor, Hariom Tower, Circular Road, Ranchi-1, Ph.: 0651-2562523, 9835508812, 8507613968 25
th
26 ( ) BY R. K. MALIK’S NEWTON CLASSES
are C11 = 5, C12 = 1, C13 = −2 , C21 = 4, C22 = 1, C23 = −2 , C31 = −2, C32 = 0, C33 = 1
 5 4 −2   5 a −2 
⇒  1 1 0  =  1 1 0 
 −2 −2 1   −2 −2 b 
On comparing the corresponding elements, we get a = 4, b = 1
∴ [a b] = [ 4 1]
1 0 0 k 0 0
175. Ans. (b), Let I =  0 1 0  , then kI =  0 k 0 
 
 0 0 1   0 0 k 
 k2 0 0 
∵ adj ( kA ) = k n −1 ( adj A )
⇒ adj ( kI ) =  0 2
0  = k 2I
  

 0 0  where k ∈ R and n is the order of matrix A


k  
 k 2 
176. Ans. (c), Since, the rank of given matrix is 1, then
2 5
=0 ⇒ 2a − 8 + 20 = 0 ⇒ a = −6
−4 a − 4
 −3   4   10 
177. Ans. (a), Given x   + y   =  
4  3   −5 
∴ −3 x + 4 y = 10 …(i) and 4 x + 3 y = −5 …(ii)
On solving equations (i) and (ii), we get x = −2, y = 1
4 1 0 0
178. Ans. (c), Let A =  3 0 1 0 
 6 0 2 0 
Now, we take a submatrix of order 3 × 3
4 1 0
B =  3 0 1  ; B = −1 ( 6 − 6 ) = 0
 6 0 2 
4 1
Now, we take a submatrix of order 2 × 2 i.e., C =  ; C = 0−3 ≠ 0
3 0
∴ Rank of matrix A is 2.
 2 4 5 
179. Ans. (b), Given A =  4
 8 10 
 −6 −12 −15 

2 4 5
⇒ A =  0 0 0  [Applying R2 → R2 − 2 R1 and R3 → R3 + 3R1 ]
 0 0 0 
Since, the equivalent matrix is echelon form has only one non-zero row.
∴ Rank ( A ) = 1

26 Office.: 606 , 6 Floor, Hariom Tower, Circular Road, Ranchi-1, Ph.: 0651-2562523, 9835508812, 8507613968
th
( ) BY R. K. MALIK’S NEWTON CLASSES 27

 1 −1 1 
180. Ans. (d), Let A =  1 1 −1 
 −1 1 1 
∴ A = 1(1 + 1) + 1(1 − 1) + 1(1 + 1) = 4 ≠ 0 . ∴ Rank of matrix A is 3.

adj ( adj A ) = A
n−2 0
A= A A= A
181. Ans. (d) A3 = A + I (given) …(i)
∵ A = A . A = ( A + I )( A + I ) = A + 2 A + I
6 3 3 2
…(ii)
∵ AI = A, I 2 = I 

A5 = A2 . A3 = A2 ( A + I ) = A3 + A2 = ( A + I ) + A2 [Using (i)]
= A2 + A + I …(iii)
∴ B = A6 − A5 = ( A2 + 2 A + I ) − ( A2 + A + I ) = A.

And so, B −1 = A−1.


182. Ans. (d), AB = A ⇒ A2 = AA = A …(i)
And BA = B ⇒ B 2 = BB = B …(ii)
∴ ( A + B ) = A2 + AB + BA + B 2 = A + B + B = 2 ( A + B ) .
2

∵ A2 = A, AB = A, BA = B, B 2 = B 

⇒ ( A + B ) = ( A + B ) . ( A + B ) = ( A + B ) .2 ( A + B )
3 2

= 2 ( A + B ) = 22 ( A + B )
2
 Using ( A + B ) 2 = 2 ( A + B ) 
 
…… ……. ……. ……...
( A + B)
7
= 26 ( A + B ) ∵ ( A + B )n = 2n −1 ( A + B ) 
 
= ( 23 ) ⇒ 82 ( A + B ) .
2
( A + B)
1 2 
183. Ans. (c), A = 
0 1 

1 2  1 2   1 4 
⇒ A2 = A. A = 
0 1  0 1  0 1 
 = 

1 4  1 2  1 6 
⇒ A3 = A2 . A = 
0 1  0 1  0 1 
 = 

….. ……. …… ……
1 2n 
.
0 1 
An = 

184. Ans. (c), Given that :


( A + B) = mC0 Am + mC1 Am −1 B + mC2 Am − 2 B 2 + ... + mCn B m …(i)
m

Putting m = 2 in (i) we get:


= 2C0 A2 + 2C1 AB + 2C2 B 2 = A2 + 2 AB + B 2 …(ii)
2
( A + B)

Office.: 606 , 6 Floor, Hariom Tower, Circular Road, Ranchi-1, Ph.: 0651-2562523, 9835508812, 8507613968 27
th
28 ( ) BY R. K. MALIK’S NEWTON CLASSES
But we know that
= ( A + B ) . ( A + B ) = A2 + AB + BA + B 2 …(iii)
2
( A + B)
From (ii) and (iii) we get:
2AB = AB + BA ⇒ AB = BA.

185. Ans. (c), A = 0 ⇒ =0 ⇒ ad − bc = 0 …(i)


a b
c d
Now, A2 − ( a + b ) A + KI = O
 a b  a b  a b 1 0 0 0
0 1 0 0
⇒   − (a + b) +k = 
 c d  c d  c d
 a 2 + bc ab + bd   a 2 + ad ab + bd   k 0   0 0 
bc + d 2   ac + cd ad + d 2   0 k   0 0 
⇒ − + = 
 ac + cd
0  0 0  k 0 0 0
[Using (i)]
 bc − ad + k
0 bc − ad + k   0 0   0 k  0 0
⇒ =  ⇒ = 

Comparing the corresponding elements of equal matrices, we get:
k = 0.
1 0  0 1   a 0  0 b   a b 
186. Ans. (c), aI + bA = a 
0 1  0 0  θ a  0 0   0 a 
 +b = + = 

a b  a b  a 2 2ab  1 0  0 1 
, a2  + 2ab  = a 2 I + 2ab A.
0 a  0 a   0 0 1  0 0 
2 
=  =  
a 
 a b c   a b c  1 0 0 
187. Ans. (d), A A = I ⇒  b c a  b c a  = 0 1 0 
T

 c a b   c a b  0 0 1 

 a 2 + b 2 + c 2 ab + bc + ca ab + bc + ca  1 0 0 
⇒  ab + bc + ca a 2 + b 2 + c 2 ab + bc + ca  = 0 1 0 
 
 ab + bc + ca ab + bc + ca a 2 + b 2 + c 2  0 0 1 
 
Equating the corresponding elements of equal matrices, we get:
a 2 + b 2 + c 2 = 1and ab + bc + ca = 0
Now, ( a + b + c ) = a 2 + b 2 + c 2 + 2 ( ab + bc + ca ) = 1 + 2. ( 0 ) = 1 ⇒ a + b + c = 1
2

∴ a 3 + b3 + c3 − 3abc = ( a + b + c ) {a 2 + b 2 + c 2 − ( ab + bc + ca )}

= 1.{1 − 0} = 1 ⇒ a 3 + b3 + c3 = 1 + 3abc = 4. [∵ abc = 1]


1 −1 a 1 
188. Ans. (b), We have: A =  , B=
 2 −1  b −1
 

1 −1  a 1   a + 1 0 
 2 −1  b −1 b + 2 −2 
∴ A+ B =  + = 

 a + 1 0   a + 1 0   a 2 + 2a + 1 0
And so, ( A + B )
2

b + 2 −2  b + 2 −2  ab + 2a − b − 2 4 
=    = 

 1 −1 1 −1  −1 0 
Also, A2 = 
 2 −1  2 −1  0 −1
 = 

28 Office.: 606 , 6 Floor, Hariom Tower, Circular Road, Ranchi-1, Ph.: 0651-2562523, 9835508812, 8507613968
th
( ) BY R. K. MALIK’S NEWTON CLASSES 29

 a 1   a 1   a 2 + b a − 1
B2 = 
 b −1  b −1  ab − b b + 1
 = 

Now, ( A + B ) = A2 + B 2
2

 a 2 + 2a + 1 0   −1 0   a 2 + b a − 1  a 2 + 2a + 1 0   a 2 + b − 1 a − 1
 ab + 2a − b − 2 4   0 −1  ab − b b + 1  ab + 2a − b − 2 4  ab − b
⇒ =
  +
   ⇒  = 
b 
Equating the corresponding elements, we get:
a 2 + 2a + 1 = a 2 + b − 1, a − 1 = 0, ab + 2a − b − 2 = ab − b, 4 = b
Solving these equations, we get : a = 1, b = 4
Another Method:
2
( A + B ) = A2 + B 2
⇒ ( A + B )( A + B ) = A2 + B 2 ⇒ A2 + AB + BA + B 2 = A2 + B 2 ⇒ AB + BA = O
1 −1  a 1   a 1  1 −1 0 0   a − b 2   a + 2 − a − 1 0 0 
 2 −1  b −1  b −1  2 −1 0 0   2a − b 3   b − 2 −b + 1 0 0 
⇒    +    =  ⇒ + = 

 2a − b + 2 − a + 1   0 0 
 2a − 2 −b + 4  0 0 
⇒ =

Equating the corresponding, we get: 2a − b + 2 = 0, − a + 1 = 0, 2a − 2 = 0, − b + 4 = 0


Solving these equations, we get: a = 1, b = 4.
189. Ans. (d), We have : AT = A, BT = B [∵ A and B are symmetric matrices]
Now, ( AB − BA ) = ( AB ) − ( BA )
T T T

= BT AT − AT BT [By Reversal Law]


= BA − AB = − ( AB − BA)  Using AT = A. BT = B 

∴ ( AB − BA ) is skew-symmetric.
190. Ans. (a), The matrix A is orthogonal since
 1 2 2   1 2 −2  9 0 0  1 0 0 
1 1 1
AAT =  2 1 −2  .  2 1 2  = 0 9 0  = 0 1 0  = I 3 .
3 3 9
   
 −2 2 −1  2 −2 −1 0 0 9  0 0 1 

b2 
191. Ans. (b), A =  2
 ab

 −a − ab 
b 2   ab b2 
A = A. A =  2
2  ab
− ab   − a 2
 
−a − ab 
 a 2b 2 − a 2b 2 ab3 − ab3  0 0 
 = O. ∴ A is a nilpotent matrix.
− a 2b 2 + a 2 b 2   0 0 
= 3 3 =
 −a b + a b
cos θ − sin θ   cos θ sin θ  cos θ − sin θ 
192. Ans. (a), We have: A =  ∴ A' =  , ( A ') ' = A =  ,
 sin θ cos θ   − sin θ cos θ   sin θ cos θ 

cos θ − sin θ
= cos 2 θ + sin 2 θ = 1.
sin θ cos θ
A =

Office.: 606 , 6 Floor, Hariom Tower, Circular Road, Ranchi-1, Ph.: 0651-2562523, 9835508812, 8507613968 29
th
30 ( ) BY R. K. MALIK’S NEWTON CLASSES
cos θ − sin θ   cos θ sin θ 
Clearly, AA ' = 
 sin θ cos θ   − sin θ cos θ 
 cos 2 θ + sin 2 θ cos θ sin θ − cos θ sin θ  1 0 
cos θ sin θ − cos θ sin θ sin 2 θ + cos 2 θ  0 1 
= = =I

∴ A is an orthogonal matrix.
If A is orthogonal matrix then A′ is always also orthogonal matrix
Determinant A = 1 ≠ 0 and so A is invertible.

193. Ans. (c), ( AB − BA ) = ( AB ) − ( BA ) = Bθ Aθ − Aθ Bθ ∵ ( XY )θ = Y θ X θ 


θ θ θ
 
= BA − AB = − ( AB − BA) ∵ Aθ = A, Bθ = B 

∴ ( AB − BA ) is a skew-Hermitian matrix.
194. Ans. (b), A is orthogonal ⇒ A AT = I
⇒ AAT = I = 1 [Taking determinant on both sides]

⇒ A =1
2
∵ AAT = A AT = A 2 since AT = A 
 
⇒ A = ±1.
∵ A is a matrix of order 3 × 3and so 
195. Ans. (d), B = 5 A = 5 A  A is a matrix of order 3 × 3and 
2 3 2  2 
 kX = k n X if X is of order n × n 
 
= 125. A = 125 × 6 × 6 = 4500.
2 2
∵ AB = A B ⇒ A2 = A 
 
196. Ans. (b), AAT = I ⇒ AAT = I

⇒ A AT = I ∵ AB = A B and I = 1

⇒ A = 1 ⇒ A = ±1
2
∵ AT = A 
 
197. Ans. (d), We have:
r −1
det ( M r ) = = r 2 − ( r − 1)
2

r −1
r
r
2008
∴ det ( M 1 ) + det ( M 2 ) + .... + det ( M 2008 ) = ∑ r 2 − ( r − 1)
2

r =1
( )
= 12 − 02 + 22 − 12 + ........ + 20082 − 2007 2 = ( 2008 ) .
2
( ) ( ) ( )
198. Ans. (c), The set of all determinants of order 2 with elements 0 and1 only, consists of 2 × 2 × 2 × 2 = 16
determinants.
These determinants are:
1 1 1 1 1 1 0 1
(i) (ii) (iii) (iv)
1 1 1 0 0 1 1 1
1 0 1 0 1 1 0 1
(v) (vi) (vii) (viii)
1 1 1 0 0 0 0 1

30 Office.: 606 , 6 Floor, Hariom Tower, Circular Road, Ranchi-1, Ph.: 0651-2562523, 9835508812, 8507613968
th
( ) BY R. K. MALIK’S NEWTON CLASSES 31
0 0 1 0 0 1 1 0
(ix) (x) (xi) (xii)
1 1 0 1 1 0 0 0
0 1 0 0 0 0 0 0
(xiii) (xiv) (xv) (xvi)
0 0 1 0 0 1 0 0
Out of these 16 determinants, the determinant (i), (vi), (vii), (viii), (ix), (xii), (xiii), (xiv) (xv) and (xvi)
are equal to zero. i.e. there are 10 determinants equal to zero; the determinants (ii), (iv) and (xi) have
negative values i.e. there are 3 determinants having negative values and the remaining 3 determinants
–(iii), (v) and (x) have positive values.
Thus, out of these 16 determinants, 13 are non-negative.
13
∴ Required probability = .
16
199. Ans. (c), We have:
DD ' = A adj A = A . A if A is a square matrix of order n 
n −1 n −1
∵ adj A = A
 
= A = Dn .
n

200. Ans. (c), We know that A. ( adj A) = A .I


Now, A ( adj A ) = 5I (given) ∴ A = 5.

Now, adj A = A , where A is a square matrix of order n 


3−1 n −1
∵ adj A = A
 
= A = 52 = 25.
2

201. Ans. (a), Since A is a square matrix of order 3, so adj A is also a square matrix of order 3.
∴ A . adj A = A
n
adj A
∵ kM = k n . M where M is a square matrix of order n 

where A is a square matrix of order n 


n −1 n −1
= A
n
A ∵ adj A = A
 
= ( −2 ) = ( −2 ) = −32. ∵ A = −2, n = 3
2 n −1 ( 2×3) −1 5
= A

202. Ans. (c), ( A + B )( A − B ) = A2 − B 2


⇒ A2 − AB + BA − B 2 = A2 − B 2 ⇒ AB = BA
2
⇒ ABA−1 = BAA−1 ⇒ ABA−1 = BI = B ⇒ ( ABA−1 ) = B 2
203. Ans. (b), Let A−1 = B
1 1
Then, det ( adj A−1 ) = det ( adj B ) = B = ∆ −2 .
n −1 2
= A−1 2
∆2
= =
A
204. Ans. (c), ( A − 2 I )( A + I ) = O

⇒ A2 + A − 2 A − 2 I = O ∵ AI = IA = A, I 2 = I 

⇒ A2 − A − 2 I = O ⇒ A−1 ( A2 ) − ( A1 A ) − 2 A−1 = O ⇒ IA − I = 2 A−1 ⇒


= A− 1
A− I
2
205. Ans. (d), The given homogeneous system of equations can be written in matrix from as

Office.: 606 , 6 Floor, Hariom Tower, Circular Road, Ranchi-1, Ph.: 0651-2562523, 9835508812, 8507613968 31
th
32 ( ) BY R. K. MALIK’S NEWTON CLASSES
 a1 b1 c1  x
AX = O where A =  a2 b2 c2  and X =  y 

 a3 b3 c3   z 
a1 b1 c1
Clearly, if A = 0i.e.if a2 b2 c2 = 0 then the system will have infinitely many solutions i.e. more
a3 b3 c3
than two solutions.
206. Ans. (b), The given system of equations can be written in matrix from as
 2 −1 −1  x 2
AX = B where A =  1 −2 1  , X =  y  , B =  −4 
   
 1 1 λ   z   4 
This system does not have unique solutions only if A = 0
2 −1 −1
i.e. if 1 −2 1 = 0 i.e. if 2 ( −2λ − 1) − ( −1)( λ − 1) − 1(1 + 2 ) = 0 i.e. if −3λ − 6 = 0i.e if λ = −2.
1 1 λ
207. Ans. (a), The given system of equation can be written in the matrix form as
 1 2 −1  x 1 
AX = B where A =  0
 0 λ + 3 , X =  y  , B =  3
  
 2λ + 1 0 1   z   0
If this system does not have unique solution then A = 0.
1 2 −1
Now, A = 0 0 λ + 3 = −2 0 − ( λ + 3)( 2λ + 1) 
2λ + 1 0 1
1
∴ A = 0 ⇒ 2 ( λ + 3)( 2λ + 1) ⇒ λ = −3or λ = − .
2
1
Thus, the system is inconsistent if λ = −3 or λ = − .
2
208. Ans. (b), The given homogeneous system of equations can be written in matrix from as
 1 −k 1  x 0 
AX = O where A =  k 3 − k  , X =  y  , O 0 
  
 3 1 −1  z  0 
The trivial solution is the only solution of this system if A ≠ 0
1 −k 1
Now, A = k 3 −k
3 1 −1
= 1. ( −3 + k ) − ( − k )( − k + 3k ) + 1. ( k − 9 ) = 2k 2 + 2k − 12 = 2 ( k − 2 )( k + 3)
∴ A ≠ 0 ⇒ 2 ( k − 2 )( k + 3) ≠ 0 ⇒ k ≠ 2, − 3 ⇒ k ∈ R − {2, − 3}
209. Ans. (a), The given homogeneous system of equations can be written in the matrix form as AX = O
where

32 Office.: 606 , 6 Floor, Hariom Tower, Circular Road, Ranchi-1, Ph.: 0651-2562523, 9835508812, 8507613968
th
( ) BY R. K. MALIK’S NEWTON CLASSES 33

1 a 0  x 0
A =  0 1 a  , X =  y  , O 0
   
 a 0 1   z  0
The system will have an infinite number of solutions if A = 0 .
1 a 0
Now, A = 0 1 a = 1(1 − 0 ) − a ( 0 − a 2 ) = 1 + a 3
a 0 1
∴ A =0 ⇒ 1 + a3 = 0 ⇒ a 3 = −1 ⇒ a = −1
Thus, the system has infinite solutions, if a = −1
210. Ans. (d), The given system of equations is x + y + z = 6 , x + 2 y + 3 z = 10 , x + 2 y + λ z = 12
1 1 1  x 6
In matrix form, it can be written as AX = B = where A = 1 2 3  , X =  y  , B = 10 
   
1 2 λ   z  12 
Clearly, the system will not have unique solution if A = 0
1 1 1 1 1 1
A = 1 2 3 = 0 1 2 [ R2 → R2 − R1 , R3 → R3 − R1 ]
1 2 λ 0 1 λ −1
= ( λ − 1) − 2 = λ − 3 ∴ A =0 ⇒ λ −3= 0 ⇒ λ =3
211. Ans. (a), The given homogeneous system of equations can be written in matrix form as AX = O where
 1 − k −1  x 0 
A =  k −1 −1 , X =  y  , O 0 
   
 1 1 −1  z  0 
For this system to have a non-zero solution, A = 0 .
1 −k −1 1 −k −1
Now, A = k −1 −1 = k −1 k −1 0 [ R2 → R2 − R1 , R3 → R3 − R1 ]
1 1 1 0 k +1 0
= ( −1)( k − 1)( k + 1) = 1 − k 2
∴ A =O ⇒ 1 − k 2 = 0 ⇒ k = −1 or k = 1 .
Hence, the given system has a non-zero solution if k = −1 or 1.
212. Ans. (d), The given homogeneous system of equations can be written in matrix form as AX = O where
 1 a −1 x 0 
A =  2 −1 a  , X =  y  , O = 0 
   
 a 1 2   z  0 
The system will be consistent if A = 0
1 −1
Now, A = 0 ⇒ 2 −1 a =0
a

a 1 2

Office.: 606 , 6 Floor, Hariom Tower, Circular Road, Ranchi-1, Ph.: 0651-2562523, 9835508812, 8507613968 33
th
34 ( ) BY R. K. MALIK’S NEWTON CLASSES
⇒ 1 ⋅ ( −2 − a ) − a ⋅ ( 4 − a 2 ) − 1 ⋅ ( 2 + a ) = 0 ⇒ a 3 − 6a − 4 = 0

2 ± 12
⇒ ( a + 2 ) ( a 2 − 2a − 2 ) = 0 ⇒ a = −2 or a = = 1± 3 .
2
Hence the given system of equations will have non-trivial solution if a = 1 ± 3 or a = −2 .

213. Ans. (a), The given equations can be written as x + 2 y − bz = 0 , 2 x + y − az = 0 , bx + ay − z = 0


The given homogeneous system of equations can be written in matrix form as AX = O where
 1 2 −b  x 0 
A =  2 1 − a  , X =  y  , O = 0 
b a −1  z  0 
This system has a non-zero solution i.e. x ≠ 0 or y ≠ 0 or z ≠ 0 which will time if A =0
1 2 −b
i.e. if 2 1 −a = 0
b a −1
i.e. if 1 ⋅ ( −1 + a 2 ) − 2 ⋅ ( −2 + ab ) − b ⋅ ( 2a − b ) = 0 i.e. if a 2 − 4ab + b 2 + 3 = 0 if a 2 + b 2 = 4ab − 3 .
214. Ans. (b), The given system of equations can be written in matrix form as AX = B where
 1 1 −2  a  0
A =  2 −3 1  , X =  b  , B =  0 
1 −5 4   c  α 
1 1 −2
Then, adj A = 2 −3 1 = 1(12 + 5 ) − 1( 8 − 1) − 2 ( −10 + 3) = 0
1 −5 4
∴ The system will be consistent if ( adj A) B = O

 −7 −7 −7   −7 6 −5   −7 6 −5  0   −5α 
T

Now, adj A =  6
 6 6  =  −7 6 −5 ∴

( adj A) B = −7 6 −5  0  =  −5α 
 −5 −5 −5   −7 6 −5  −6 6 −5 α   −5α 
Hence the system of equations will be consistent if α = 0
215. Ans. (c), The given homogeneous system of linear equations is x + 4ay + az = 0 , x + 3by + bz = 0
x + 2cy + cz = 0
1 4a a 
The coefficient matrix is A = 1 3b b 
1 2c c 
1 4a a
Clearly, the system has a non-trivial solution only if A = 1 3b b is equal to zero.
1 2c c
1 4a a 0 4a − 2c a − c
Now, A = 1 3b b = 0 3b − 2c b−c [ R1 → R1 − R3 , R2 → R2 − R3 ]
1 2c c 1 2c c

34 Office.: 606 , 6 Floor, Hariom Tower, Circular Road, Ranchi-1, Ph.: 0651-2562523, 9835508812, 8507613968
th
( ) BY R. K. MALIK’S NEWTON CLASSES 35
= ( 4a − 2c )( b − c ) − ( 3b − 2c )( a − c ) = ab − 2ac + bc
2 1 1
∴ A =0 ⇒ ab − 2ac + bc = 0 ⇒ 2ac = ab + bc ⇒ = + ⇒ a, b, c are in H.P.

Thus, if the given system of linear equations has a non-trivial solution then a, b, c are in H.P.
b a c

216. Ans. (a), The given system of linear equations can be written in matrix form as AX = B where
1 2 −3  x 1 
A = 0
 0 p + 2  , X =  y  , B =  3 
  
0 2 p + 1 1   z   2 
This system has no solution if A = 0 and ( adj A) B ≠ O
1 2 −3
−1
Now, A = O ⇒ 0 0 p + 2 = 0 ⇒ − ( p + 2 )( 2 p + 1) = 0 ⇒ p = −2 or p =
2
0 2 p +1 1
 − ( p + 2 )( 2 p + 1) 0 0   − ( p + 2 )( 2 p + 1) −6 p − 5 2 p + 4 
and, adj A  −6 p − 5 1 −2 p − 1 0 1 − p − 2
   
=
 2p+4 −p−2 0   0 −2 p − 1 0 

 − ( p + 2 )( 2 p + 1) −6 p − 5 2 p + 4  1 
0 1 − p − 2   3 
 
∴ ( adj A ) B = 
 0 −2 p − 1 0   2 

 −2 p 2 − 19 p − p   − ( 2 p + 1)( p + 9 ) 
1
−2 p − 1  =  − ( 2 p + 1)  for p = −2 or −
  
2

= ∴ ( adj A) B ≠ O

 −6 p − 3   −3 ( 2 p + 1) 
  
Hence, the given system has no solution if p = −2 .
217. Ans. (c), The given system of equations can be written in matrix form as AX = B where
1 1 1 x 6
A = 1 2 3  , X =  y  , B = 10 
  
1 2 λ   z   µ 
This system has no solution (i.e. it is inconsistent) if A = 0 and ( adj A) β ≠ O .
1 1 1
Now, A = 0 ⇒ 1 2 3 =0
1 2 λ
⇒ 1( 2λ − 6 ) − 1( λ − 3) + ( 2 − 2 ) = 0 ⇒ λ =3

1 1 1 0 0 0  0 −1 1 
T

using λ = 3 , we get : A = 1 2 3 ∴ adj A =  −1 2 −1 = 0 2 −2 


1 2 3  1 −2 1  0 −1 1 

0 −1 1   6   −10 + µ 
And so, ( adj A) B = 0 2 −2 10 =  20 − 2µ 
0 −1 1   µ   −10 + µ 

Office.: 606 , 6 Floor, Hariom Tower, Circular Road, Ranchi-1, Ph.: 0651-2562523, 9835508812, 8507613968 35
th
36 ( ) BY R. K. MALIK’S NEWTON CLASSES
 −10 + µ  0 
Now, ( adj A) B = O if  20 − 2 µ  = 0  i.e. µ = 10 . ∴ ( adj A) B ≠ O if µ ≠ 10 .
 −10 + µ  0 
This, given system of equations has no solution if λ = 3 and µ ≠ 10 .
218. Ans. (c), The given system of equations can be written as
x − cy − bz = 0 , cx − y + az = 0 , bx + ay − z = 0
This is a homogeneous system of equations which in matrix form can be written as:
1 −c −b x 0
AX = O where A = c −1 a , X =  y  , O 0
 
b a 1  z  0
This system will have a non-trivial solution i.e. a solution other than x = 0, y = 0, z = 0 , if A = 0 .
1 −c −b 1 0 0
Now, A = c −1 a = c c 2 − 1 a + bc [C2 → C2 + c.C1 , C3 → C3 + b.C1 ]
b a −1 b a + bc b 2 − 1
= ( c 2 − 1)( b 2 − 1) − ( a + bc )( a + bc ) = ( b 2 c 2 − b 2 − c 2 + 1) − ( a 2 + 2abc + b 2 c 2 )

= − ( a 2 + b 2 + c 2 + 2abc − 1)

∴ A =0 ⇒ a 2 + b 2 + c 2 + 2abc = 1 , which is the required condition.


1 1 0
219. Ans. (b), Let A = 1 1 0
1 1 0

1 1 1   1 1 1 
Then, A = 1 1 1  ~  0 0 0 
T
[ R2 → R2 − R1 ]
0 0 0  0 0 0 

∴ ρ ( A ) = ρ ( AT ) = 1
Another Method:
1 1 0  1 1 0 
Let A = 1 1 0  − 0 0 0  [ R2 → R2 − R1 , R3 → R3 − R1 ]
1 1 0  0 0 0 
This matrix A has been reduced to the Echelon form. ∴ ρ ( A) = number of non-zero rows = 1 .
 1 −1 1 
220. Ans. (d), Let A =  1 1 −1
 −1 1 1 

 1 −1 1 
~  0 2 −2  [ R2 → R2 − R1 , R3 → R3 + R1 ]
 0 0 2 
This matrix is in Echelon form.
∴ ρ ( A) = 3 .

36 Office.: 606 , 6 Floor, Hariom Tower, Circular Road, Ranchi-1, Ph.: 0651-2562523, 9835508812, 8507613968
th
( ) BY R. K. MALIK’S NEWTON CLASSES 37

 1 2 −1
221. Ans. (d), A =  −1 1 2 
 2 −1 1 

1 2 −1
~ 0 3 1  [ R2 → R2 + R1 , R3 → R3 − 2R1 ]
0 −5 2 

 1 −1 2 
~ 0 1 3  [C2 ↔ C3 ]
0 3 −5
 1 −1 2 
~  0 1 3  [ R3 → R3 − 3R2 ]
 0 0 −14 
This matrix is in echelon form: ∴ ρ ( A) = 3 .
2 4 5 
222. Ans. (b), A =  4
 8 10 
 −6 −12 −15

2 4 1
~4
 8 2  [C3 → C3 − 2C1 ]
 −6 −12 −3
1 4 2
~  2 8 4  [C2 ↔ C3 ]
 −3 −12 −6

1 4 2
~ 0 0 0  [ R2 → R2 − 2 R1 , R3 → R3 + 3R1 ]
0 0 0 
This matrix is in Echelon form, in which the number of non-zero rows = 1 .
∴ ρ ( A) = 1
 −1 2 5 
223. Ans. (a), Let A =  2 −4 a − 4

 1 −2 a + 1 

1 −2 −5 
~  2 −4 a − 4 [ R1 → − R1 ]
1 −2 a + 1 
 1 −2 −5 
~ 0 0 a + 6  [ R2 → R2 − 2 R1 , R3 → R3 − R1 ]
0 0 a + 6 

Office.: 606 , 6 Floor, Hariom Tower, Circular Road, Ranchi-1, Ph.: 0651-2562523, 9835508812, 8507613968 37
th
38 ( ) BY R. K. MALIK’S NEWTON CLASSES
 1 −2 −5 
~ 0 0 a + 6  [ R3 → R3 − R2 ]
0 0 0 
Clearly, ρ ( A) = 2 if a + 6 ≠ 0 i.e. if a ≠ −6 i.e. if a = 1 (in particular).
And ρ ( A) = 1 if a + 6 = 0 i.e. if a = −6
Under no circumstances, we can have ρ ( A) = 3 or ρ ( A) = 4 .
224. Ans. (b), The given system of equations can be written in matrix form as
1 1 1  x 1
AX = B where A = 1 2 4  , X =  y  , B =  α 
1 4 10   z  α 2 

1 1 1 : 1 
We have : ( A : B ) = 1 2 4 : α 
1 4 10 : α 2 

1 1 1 : 1 
~  0 1 3 : α − 1

[ R2 → R2 − R1 , R3 → R3 − R1 ]
 0 3 9 : α − 1

1 1 1 : 1 
~ 0 1 3 :
 α − 1  [ R3 → R3 − 3R2 ]
0 0 0 : α − 3α + 2
2

Clearly, ρ ( A) = 2 .
The system will be consistent if ρ ( A : B ) = ρ ( A ) = 2 .
i.e. if α 2 − 3α + 2 = 0 i.e. if (α − 1)(α − 2 ) = 0 ⇒ α = 1 or 2
3 4 5 6 7
4 5 6 7 8
225. Ans. (b), Let A = 
5 6 7 8 9

10 11 12 13 14 
 

3 4 5 6 7
1 1 1 1 1 
~
2
[ R2 → R2 − R1 , R3 → R3 − R1 , R4 → R4 − R1 ]
2 2 2 2
7 7 7 7 7
 

1 1 1 1 1
3 4 5 6 7 
~
2 2 2 2 2
[ R1 ↔ R2 ]
7 7 7 7 7
 

1 1 1 1 1
0 1 2 3 4 
~
0
[ R2 → R2 − 3R1 , R3 → R3 − 2 R1 , R4 → R4 − 7 R1 ]
0 0 0 0
0 0 0 0 0
 

38 Office.: 606 , 6 Floor, Hariom Tower, Circular Road, Ranchi-1, Ph.: 0651-2562523, 9835508812, 8507613968
th
( ) BY R. K. MALIK’S NEWTON CLASSES 39
This matrix is in the Echelon Form.
∴ ρ ( A) = 2 .
226. Ans. (c), The characteristic equation of the matrix A is A − λ I 3 = 0
1− λ 1 0
i.e. 1 2−λ 1 =0 i.e. (1 − λ ) ( λ 2 − 2λ − 1) − 1( −λ − 2 ) = 0
2 1 −λ
i.e. λ 3 − 3λ 2 − 1 = 0 …(i)
By Cayley − Hamilton Theorem, every matrix satisfies its characteristic equation.
∴ Matrix A must satisfy equation (i). So, we have :
A3 − 3 A2 − I = O
227. Ans. (a), The characteristic equation of the matrix A is A − λ I 2 = 0
−3 − λ 4
i.e. = 0 i.e. ( 3 + 4λ + λ 2 ) − 8 = 0
2 −1 − λ
i.e. λ 2 + 4λ − 5 = 0 i.e. ( λ + 5 )( λ − 1) = 0 i.e. λ = −5 or λ = 1
∴ The characteristic roots of matrix A are −5 and 1.
228. Ans. (c), The characteristic equation of the matrix A is A − λ I = 0
5−λ 4
i.e. = 0 i.e. λ 2 − 7λ + 6 = 0
1 2−λ
Solving this characteristic equation, we get : λ 2 − 7λ + 6 = 0 ⇒ ( λ − 1)( λ − 6 ) = 0 ⇒ λ = 1, λ = 6
Thus, characteristic roots or eigen values of matrix A are 1 and 6.
1 0 0 
229. Ans. (b), Let A =  2 3 0 
 4 5 6 

The characteristic equation of the matrix A is A − λ I 3 = 0


1− λ 0 0
⇒ 2 3−λ 0 = 0 i.e. (1 − λ )( 3 − λ )( 6 − λ ) = 0
4 5 6−λ
⇒ 1 − λ = 0 or 3 − λ = 0 or 6 − λ = 0 ⇒ λ = 1 or λ = 3 or λ = 6
∴ The characteristic roots of matrix A are 1, 3 and 6.
230. Ans. (a), The characteristic equation of the matrix A is A − λ I 2 = 0
1− λ 3
i.e. = 0 i.e. (1 − λ )( 5 − λ ) − 12 = 0 i.e. λ 2 − 6λ − 7 = 0 ..(i)
4 5−λ
By Cayley-Hamilton Theorem, every matrix satisfies its characteristic equation.
∴ Matrix A must satisfy equation (i). So, we have: A2 − 6 A − 7 I = O
Multiplying by A−1 on both sides, we get:
A−1 ( A2 ) − 6 ( A−1 A ) − 7 A−1 = O [∵ XI = X , XO = O ]

Office.: 606 , 6 Floor, Hariom Tower, Circular Road, Ranchi-1, Ph.: 0651-2562523, 9835508812, 8507613968 39
th
40 ( ) BY R. K. MALIK’S NEWTON CLASSES
⇒ ( A A ) A − 6 ( A A ) − 7 A−1 ⇒ A − 6 I = 7 A−1
−1 −1
∵ A−1 A = I 

1
⇒ A−1 = ( A − 6I )
7
231. Ans. (d), The Chatacteristic equation of matrix A is A − λ I 3 = 0

1− λ 1 0
i.e. 0 1− λ 1 = 0 i.e. (1 − λ ) = 0
3

0 0 1− λ

i.e. 1 − λ 3 − 3λ + 3λ 2 = 0 i.e. λ 3 − 3λ 2 + 3λ − 1 = 0 …(i)


By Cayley-Hamilton theorem, every matrix satisfies its characteristic equation.
∴ Matrix A must satisfy equation (i).
So, we have : A2 − 3 A2 + 3 A − I 3 = O
Multiplying by A−1 on both sides, we get:
A−1 ( A3 ) − 3 A−1 ( A2 ) + 3 A−1 A − A−1 I 3 = A−1O [Using Associative Law, XI = X , XO = O ]

⇒ IA2 − 3IA + 3I 3 − A−1 = O ∵ A−1 A = I 

⇒ A2 − 3 A + 3I 3 − A−1 = O ⇒ A−1 = A2 − 3 A + 3I 3
 a +1 0 
232. Ans. (b) We have, A + B = 
 b + 2 −2 

 1 −1 1 −1  −1 0   a 1   a 1   a 2 + b a − 1
A2 =  and 2
 b −1  b −1 = 
 2 −1  2 −1  0 −1   ab − b b + 1
 =
   B = 
 
 a + 1 0   a + 1 0   ( a + 1) 0
2

than ( A + B )
2 2
∴ ( A + B ) = A2 + B 2
b + 2 −2  b + 2 −2  ( a − 1)( b + 2 ) 4 
=   =  

 ( a + 1)2 0   −1 0   a 2 + b a − 1
( a − 1)( b + 2 ) 4   0 −1  ab − b b + 1
⇒ = + 

 ( a + 1)2 0   a 2 + b − 1 a − 1
⇒ a = 1, b = 4
( a − 1)( b + 2 ) 4   ab − b
⇒ = 
b 

233. Ans. (a) we have, AB = A B ≠ 0 ∵ A ≠ 0, B ≠ 0 

Therefore, AB is a non-singular matrix. Consequently, ( AB ) exists and ( AB ) = B −1 A−1.


−1 −1

234. Ans. (a), It is given that A ≠ 0. So, A−1 exists.

Now, AB = O ⇒ A−1 ( AB ) = A−1O ⇒ ( A−1 A ) B = O ⇒ IB = O ⇒ B=O


1 + 2i 
235. Ans. (d) We have, A = 
 −i
 −1 + 2i 0 
−1 + 2i  1 − 2i 
= − A . Hence, A is skew-hermitian.
 −i  i
1 + 2i 0   −1 − 2i 0 
( A)
T
=  = −

236. Ans. (c), Clearly, the element in the first row and third column of the inverse of the given matrix is
cofactor of the element in third row and first column of the given matrix.

40 Office.: 606 , 6 Floor, Hariom Tower, Circular Road, Ranchi-1, Ph.: 0651-2562523, 9835508812, 8507613968
th
( ) BY R. K. MALIK’S NEWTON CLASSES 41
2 −3
Hence, required element = = 7 . Note that determinant of the matrix has value one
1 2
237. Ans. (b) Since A is an orthogonal matrix.
 1 2 2   1 2 a  1 0 0 
1 1
⇒  2 1 −2 ⋅  2 1 2  = 0 1 0
3 3
∴ AA = I
T

 a 2 b   2 −2 b  0 0 1 

 1 2 2   1 2 a  1 0 0  9 0 a + 4 + 2b  9 0 0
1

⇒  2 1 −2  2 1 2  = 0 1 0  0 9 2a + 2 − 2b  = 0 9 0
9

⇒
 a 2 b   2 −2 b  0 0 1   a + 4 + 2b 2a + 2 − 2b a 2 + 4 + b 2  0 0 9 
⇒ a + 4 + 2b = 0, 2a + 2 − 2b = 0 and a 2 + 4 + b 2 = 9
⇒ a + 2b + 4 = 0, a − b + 1 = 0 and a 2 + b 2 = 5 ⇒ a = −2, b = −1.
0 1 −1
238. Ans. (a), We have, A = 2 1 3 = 0 + 7 −1 = 6
3 2 1
∴ ( A ( adj A ) A−1 ) A = ( A ( adj A ) ) ( A−1 A ) ⇒ ( A ( adj A ) A−1 ) A = ( A I ) I

6 0 0 3 0 0
⇒ ( A ( adj A ) A ) A = A I = 0 6 0 = 2 0 3 0  . Hence, option (a) is correct.
−1  
0 0 6 0 0 3
239. Ans. (b) We have, A an orthogonal matrix ⇒ A = ± 1
B a skew-symmetric matrix of odd order ⇒ B = 0
∴ AB = A B ⇒ AB = ( ±1) × 0 = 0.
240. Ans. (d) It is given that B is the inverse of matrix A.
1 −1 1   4 2 2  10 0 0 
∴ AB = I ⇒ A (10 B ) = 10 I ⇒  2 1 −3  −5 0 α  =  0 10 0 
1 1 1   1 −2 3   0 0 10 

10 0 5 − α  10 0 0 
⇒  0 10 α − 5 =  0 10 0  ⇒α =5
 0 0 α + 5  0 0 10 
241. Ans. (a) We have, A2 = A and B = I − A
2
∴ AB + BA + I − ( I − A ) = A ( I − A ) + ( I − A) A + I − ( I − A )( I − A)
= A − A 2 + A − A2 + I − ( I − 2 A + A 2 ) = A − A + A − A + I − ( I − 2 A + A ) ∵ A2 = A
=A
2
n −1
242. Ans. (c) We know that adj ( adj A ) = A ( ) .

So, statement-2 is false. If A is an orthogonal matrix, then A = ±1


2 2
n −1 n −1
∴ adj ( adj A ) = A ( ) = ( ±1)( ) = ±1

243. Ans. (b), AAT = I ⇒ ( A−1 A ) AT = A−1.I ⇒ I . AT = A−1 ⇒ A−1 = AT .

Office.: 606 , 6 Floor, Hariom Tower, Circular Road, Ranchi-1, Ph.: 0651-2562523, 9835508812, 8507613968 41
th
42 ( ) BY R. K. MALIK’S NEWTON CLASSES
244. Ans. (b), We have, ( AB − BA ) = ( A B ) − ( BA ) = BT AT − AT BT
T T T

⇒ ( A B − B A ) = BA − AB ∵ AT = A, BT = B 
T

⇒ ( AB − BA ) = − ( AB − BA ) . So, AB − BA is skew-symmetric matrix.


T

245. Ans. (b), We know that A ( adj A) = A I n


Now, A is singular ⇒ A = 0
∴ A ( adj A ) = A I n ⇒ A ( adj A ) = OI n = O . Hence, A ( adj A) is the null matrix.
246. Ans. (b), We know that A ( adj A) = A I
 cos x sin x 
If, A =   , then A = 1. ∴ A ( adj A ) = I ⇒ kI . = I ⇒ k = 1.
 − sin x cos x 
247. Ans. (a), We have, A =  aij  ∴ k A = kn A .

 1 − tan θ   1 tan θ 
−1

248. Ans. (b), We have, 


 a −b 
 tan θ 1   − tan θ 1   = 
b a 
 1 − tan θ  1  1 − tan θ   a −b 
 tan θ 1  1 + tan 2 θ
⇒   tan θ 1   b a 
=

1 − tan 2 θ −2 tan θ 
1 1 − tan 2 θ −2 tan θ   a −b  1 + tan 2 θ 1 + tan 2 θ   a −b 
1 + tan 2 θ  2 tan θ 1 − tan 2 θ   b a   2 tan θ 1 − tan 2 θ   b a 
⇒   = ⇒  = 
1 + tan 2 θ 1 + tan 2 θ 
cos 2θ − sin 2θ   a −b 
⇒ a = cos 2θ , b = sin 2θ
 sin 2θ cos 2θ   b a 
⇒ =

249. Ans. (d), We have, 3 A3 + 2 A2 + 5 A + I = 0


⇒ I = −3 A3 − 2 A2 − 5 A ⇒ IA−1 = ( −3 A3 − 2 A2 − 5 A ) A−1 ⇒ A−1 = −3 A2 − 2 A − 5 I
250. Ans. (d), Inverse matrix if it exist is always unique.
Since, A is invertible. Therefore, A ≠ 0. Thus, option (d) is correct.
 −i 0   −i 0   −i 0   −1 0 
251. Ans. (b), We have, A =  ∴ A2 = 
 0 −i   0 −i   0 −i   0 −1
  = 

⇒ A2 = − I ⇒ A2 + I = O
252. Ans. (a) Let A be a skew-symmetric matrix of odd order ( 2n + 1) say. Since A is skew-symmetric.

⇒ AT = ( −1)
2 n +1
∴ AT = − A ⇒ AT = − A A ⇒ AT = − A

⇒ A =− A ⇒ 2 A =0 ⇒ A =0
253. Ans. (b) Since, A is orthogonal matrix.
∴ A AT = I = AT A ⇒ A AT = I = AT A ⇒ A AT = 1 = AT A

=1 ⇒ A = ±1.
2
⇒ A
254. Ans. (a), Since, A is non-singular matrix. ∴ A ≠ 0 ⇒ rank ( A ) = n
255. Ans. (b), Let, A denote the matrix every element of which is unity. Then, all the 2-rowed minors of A
obviously vanish. But A is a non-null matrix. Hence, rank of A is 1.

42 Office.: 606 , 6 Floor, Hariom Tower, Circular Road, Ranchi-1, Ph.: 0651-2562523, 9835508812, 8507613968
th
( ) BY R. K. MALIK’S NEWTON CLASSES 43
1 2 3 0
2 4 3 2 
256. Ans. (c), Let A =  .
3 2 1 3
6 8 7 5
 

1 2 3 0
2 4 3 2 
Then, A ∼  By applying R4 → R4 − R3 − R2 − R1
3 2 1 3
0 0 0 0
 

1 2 3 0
 0 0 −3 2 
By applying R2 → R2 − 2 R1 , R3 → R3 − 3R1
 0 −4 −8 3
⇒ A∼ 

0 0 0 0
 

1 2 3 0
0 −4 −8 3
⇒ A~
0 0 −3 2
0 0 0 0
Clearly if is in Echlon form which has three non-zero rows
2 3 1 4 
257. Ans. (a), We have, A =  0 1 2 −1
 0 −2 −4 2 

 2 3 1 4
⇒ A ∼  0 0 0 0  Applying R2 → 2 R2 + R3
 0 −2 −4 2

 2 3 −5 7 
C3 → C3 − 2C2
⇒ A ∼  0 0 0 0  Applying
 0 −2 0 0 
C4 → C4 + C2

 2 3 −5 7 
⇒ A ∼  0 −2 0 0  Applying R2 ↔ R3
 0 0 0 0 
Which is now in echlon form with two non-zero rows
x2 y2 z2
258. Ans. (d) Let 2 = X , 2 = Y and 2 = Z .

Then the given system of equations reduces to X + Y − Z = 1, X − Y + Z = 1, − X + Y + Z = 1.


a b c

 1 1 −1
The coefficient matrix A of the above system of equation is given by A =  1 −1 1  .
 −1 1 1 
Clearly, A ≠ 0 , So, the given system of equations has a unique values of x, y, z
But for every value of x there are two values of x similarly for every of y and z there will be two values
each of y and z
Hence there will be eight ordered triplets ( x, y, z )

Office.: 606 , 6 Floor, Hariom Tower, Circular Road, Ranchi-1, Ph.: 0651-2562523, 9835508812, 8507613968 43
th
44 ( ) BY R. K. MALIK’S NEWTON CLASSES
Which will satisfy the given system of equations
∴ correct choice is (d)
259. Ans. (b), We have, A = −1, B = 3 . ∴ 3 AB = 33 AB = 33 A B = 33 × ( −1) × 3 = −81 .
260. Ans. (a) It is given that the point ( x1 , y1 ) , ( x2 , y2 ) and ( x3 , y3 ) are collinear.
x1 y1 1
Therefore, x2 y2 1 = 0 . Hence, the rank of the given matrix is less than 3.
x3 y3 1

261. Ans. (d), We know that Adj ( Adj A ) = A A if A = 0 [if order of A = n ]


n−2

⇒ Adj ( Adj A) = A . A [Here n = 3 ]

⇒ det Adj ( Adj A ) = A .det A = A = (14 ) (given) ⇒ A = 14.


3 4 4

Now, A = x (1 + 2 ) − 2 ( −1 − 4 ) − 1(1 − 2 ) = 14 ⇒ 3 x + 11 = 14 ⇒ x = 1.
262. Ans. (c), AB = AC ⇒ A ( B − C ) = 0, which is possible without being A = 0 or B − C = 0.
Hence (a) and (b) are not correct choice.
Further det  A ( B − C )  = 0 ⇒ det A.det ( B − C ) = 0 ⇒ det A = 0 or det ( B − C ) = 0 .
Hence, option (c) is correct.
263. Ans. (b), Skew symmetric matrix of odd order is always singular and hence non-invertible.
264. Ans. (a), A = 10
1 −1
cofactor of ( 3, 2 ) entry of given matrix 0 1
th

⇒ 10a23 = −1
10
a23 = ⇒ a23 =
A
265. Ans. (b), C = A = 3 .
 7 −10 17 
266. Ans. (b), 3 A + 4 B′ =  …(i)
 0 6 31 

 −1 18 
2 B − 3 A′ =  4 −6  …(ii)
 −5 −7 

 7 0   7 0
Taking transpose of (i) so ( 3 A + 4 B′ ) =  −10 6  ⇒ 3 A′ + 4 B =  −10 6 
′   
 17 31   17 31

 −1 18   6 18 
Now solving it with 2 B − 3 A′ =  4 −6  we get B =  −6 0 
 
 −5 −7  12 24

267. Ans. (b), If A is a third order matrix then, adj A = A …(i)


2

Given that adj A = 28561 ⇒ A = 28561 (using (i)) ⇒ A = ±13


2 4

268. Ans. (d), A ( adj A) = A I n

44 Office.: 606 , 6 Floor, Hariom Tower, Circular Road, Ranchi-1, Ph.: 0651-2562523, 9835508812, 8507613968
th
( ) BY R. K. MALIK’S NEWTON CLASSES 45
1 0 0
A ( adj A ) = 4 0 1 0 ∴ A = 4, n = 3
0 0 1

adj ( adj A ) = A = 42 = 44 = 256


( n −1)2 2

adj ( adj A ) 256


adj A = A = 4 2 = 16 = 16 .
n −1

16
⇒ =
A
β  α β  α 2 + βγ αβ + βδ  1 0 
269. Ans. (a), Here 
α
δ   γ δ  αγ + δγ βγ + δ 2  0 1 
= =
γ
⇒ α 2 + βγ = 1, βγ + δ 2 = 1
β (α + δ ) = 0 = γ (α + δ ) ⇒ α 2 = δ 2 . so that δ = α , β = 0 = γ and α = δ .
270. Ans. (a), Since, x = y = 1 is solution.
∴ a2 − a = 1 − a and b + ( 3 − 2b ) = 3 + a
⇒ a = ±1 and −b = a ….(1)
Also, equation has unique solution.
 a2
 ≠ 0 ⇒ a ( 3 − 2b ) + ab ≠ 0 ⇒ a ( 3a − 2ab + b ) ≠ 0
−a  2

3 2
⇒ 
 b − b 
∴ a ≠ 0 and 3a + b ≠ 2ab
From equation (i), if a = 1, b = −1 ⇒ 3a + b ≠ 2ab
But if a = −1, b = 1 ⇒ 3a + b = 2ab (which is not possible). ∴ ( a = 1, b = −1) is only solution.
271. Ans. (a), The augmented matrix C = [ A B ]
1 1 1 8  1 1 1 8 
= 1 −1 2 6 ∼ 0 −2 1
   −2  [ R2 → R2 − R1 , R3 → R3 − 3R1 ]
 3 5 −7 14   0 2 −10 −10 
   

1 1 1 8 
=  0 −2 1 −2  [ R3 → R3 + R2 ]
 0 0 −9 −12 
 

Rank of A = 3, rank of C = 3 . So, rank of A = rank of C = 3 = number of unknowns.


272. Ans. (a), We have, AB = BA = B ' A ' = ( AB ) ' ⇒ AB is symmetric
Also, ABA−1 = BAA−1 = B ⇒ A−1 ABA−1 = A−1 B ⇒ BA−1 = A−1 B
⇒ ( A−1 B ) ' = ( BA−1 ) ' = ( A−1 ) ' B ' = A−1 B . Since, A−1 is symmetric

273. Ans. (b), Given, AB = 3I ⇒ A−1 ( AB ) = A−1 ( 3I ) ⇒ B = 3 A−1 ⇒ A−1 =


3
B

1 1 1   x   6 
274. Ans. (b), The given system of equations may be written as 1 2 3   y  = 10 
1 2 λ   z   µ 
    

1 1 1   x   6
Applying R2 → R2 − R1 , R3 → R3 − R1

⇒ 0 1
 2   y = 4
  
 0 1 λ − 1  z   µ − 6 
    

Office.: 606 , 6 Floor, Hariom Tower, Circular Road, Ranchi-1, Ph.: 0651-2562523, 9835508812, 8507613968 45
th
46 ( ) BY R. K. MALIK’S NEWTON CLASSES
1 1 1   x  6 
⇒ 0 1
 2   y  = 4
     Applying R3 → R3 − R2
0 0 λ − 3  z   µ − 10

For λ = 3 and µ ≠ 10, we observe that the rank of the coefficient matrix is 2 and that of the augmented
matrix is 3. So, the given system of equations has no solution.
275. Ans. (b) We have, ( I − A ) ( I + A + A2 ) = I + A + A2 − A − A2 − A3 = I − O = I
−1
⇒ ( I − A ) = I + A + A2
276. Ans. (a), Let A be a symmetric matrix. Then, AA−1 = I
−1 −1
⇒ ( A A− 1 ) = I ⇒ ( A−1 ) AT = I ⇒ ( A−1 ) = ( AT ) ⇒ ( A−1 ) = ( A )
T T T T
∵ AT = A 

⇒ A−1 is a symmetric matrix


277. Ans. (a), Since A is symmetric. Therefore, AT = A
Now, ( An ) = ( AT ) = ( A ) . ∴ An is also a symmetric matrix.
T n n

278. Ans. (d), It is given that A is a skew-symmetric matrix.


 An if n is even
⇒ ( An ) = ( −1) An
− A if n is odd
⇒ ( AT ) = ( − A ) ⇒ ( An ) =  n
n n T n T
∴ AT = − A

Hence, An is skew- symmetric when n is odd.


279. Ans. (a), ( A − λ I )( B − λ I ) = ( B − λ I )( A − λ I ) ⇒ AB − λ ( A + B ) I + λ I 2 ⇒ AB = BA
1  1  1  1 
280. Ans. (b),  A '− I  A − I  = I and  A '+ I   A + I  = I
2  2  2  2 
 
 
( Subtracting the two results ) ⇒ A + A ' = 0
1  1  1  1  1
⇒ A' = −A Then  A′ − I   A − I  = I ⇒  − A − I   A − I  = I ⇒ A2 − I = − I
2  2  2  2  4
 
 
3  −3 
⇒   = ( det ( A ) ) ⇒ n is even ∴ ( det A ) is + ve
2 2 2
n

4  4 
A =− I

281. Ans. (b), symmetric means A′ = A skewsymmetric means A′ = − A


Hence 2 A = 0 ⇒ A = 0

282. Ans. (a), ∵ A =   = (Hermitian matrix ) + (skew-Hermitian matrix ).


 A + Aθ   A − Aθ 
 2   2 
+

283. Ans. (c), = A2 − AB − BA + B 2 = A2 − 2 AB + B 2


2
( A − B) (∵ AB = BA)
284. Ans. (b), Since B = A−1 BA ⇒ AB = BA
Also AB = − BA ∴ − BA = AB i.e., BA = 0
⇒ B = 0 (∵ A−1 exists) ∴ ( A + B ) = A2
2

285. Ans. (c), AB = −2 BA and AB = BA ⇒ B = 0 ∴ ( A + B ) = A3 .


3

46 Office.: 606 , 6 Floor, Hariom Tower, Circular Road, Ranchi-1, Ph.: 0651-2562523, 9835508812, 8507613968
th
( ) BY R. K. MALIK’S NEWTON CLASSES 47
Assertion reason Type
286. Ans. (c), A satisfies A2 − Tr ( A ) . A + ( det A ) I = 0
On comparing with A2 − I = 0, we get Tr ( A) = 0, A = −1
Alternate :

Let A =   , a , b, c , d ≠ 0
a b 
c d 
 a 2 + bc
Now A2 =  . c d
2
⇒ a 2 + bc = 1, bc + d 2 = 1
a b  a b  ab + bd 
2 
 ⇒ A = 
c d     ac + cd bc + d 
and ab + bd = ac + cd = 0
Also, c ≠ 0 and b ≠ 0 ⇒ a + d = 0
Tr ( A) = a + d = 0 and A = ad − bc = −a 2 − bc = −1
287. Ans. (d), A−1 exist only for non singular matrix
AB = AC ⇒ A−1 ( AB ) = A−1 ( AC )

⇒ ( A−1 A ) B = ( A−1 A ) C ⇒ −1
( A A) B = ( A A) C −1

⇒ IB = IC ⇒ B = C , if A−1 exist.
∴ A ≠0 ⇒ Statement-1 is false and statement-2 is true.
288. Ans. (a), Statement-1 is true.
adj ( adj A ) = adj ( adj A ) = A
n−2 ( n − 2) n n 2 − 2 n +1 ( n −1)2
∴ A A = A A ⇒ = A

289. Ans. (c), We have know that adj ( adj A ) = A


( n −1)2

So, Statement-2 is false.


If A is an orthogonal matrix, then A = ±1

adj ( adj A ) = A = ( ±1) = ±1


( n −1)2 ( n −1)2

290. Ans. (a), adj A = A = A ,


n −1 2 −1
= A

Office.: 606 , 6 Floor, Hariom Tower, Circular Road, Ranchi-1, Ph.: 0651-2562523, 9835508812, 8507613968 47
th

Das könnte Ihnen auch gefallen